NCLEX - Cardiovascular

अब Quizwiz के साथ अपने होमवर्क और परीक्षाओं को एस करें!

A client is wearing a continuous cardiac monitor, which begins to sound its alarm. The nurse sees no electrocardiographic complexes on the screen. Which is the priority nursing action? 1.Call a code. 2.Call the primary health care provider. 3.Check the client's status and lead placement. 4.Press the recorder button on the electrocardiogram console.

3.Check the client's status and lead placement. Rationale:Sudden loss of electrocardiographic complexes indicates ventricular asystole or possibly electrode displacement. Accurate assessment of the client and equipment is necessary to determine the cause and identify the appropriate intervention. The remaining options are secondary to client assessment.

The nurse is assessing an electrocardiogram (ECG) rhythm strip for a client. The PP and RR intervals are regular. The PR interval is 0.14 second, and the QRS complexes measure 0.08 second. The overall heart rate is 82 beats/min. The nurse should report the cardiac rhythm to be which rhythm? 1.Sinus bradycardia 2.Sick sinus syndrome 3.Normal sinus rhythm 4.First-degree heart block

3.Normal sinus rhythm Rationale:Normal sinus rhythm is defined as a regular rhythm with an overall rate of 60 to 100 beats/min. The PR and QRS measurements are normal, measuring 0.12 to 0.20 second and 0.04 to 0.10 second, respectively.

The home health nurse makes a home visit to a client who has an implanted cardioverter-defibrillator (ICD) and reviews the instructions concerning pacemakers and dysrhythmias with the client. Which client statement indicates that further teaching is necessary? 1."If I feel an internal defibrillator shock, I should sit down." 2."I won't be able to have a magnetic resonance imaging test (MRI)." 3."My wife knows how to call the emergency medical services (EMS) if I need it." 4."I can stop taking my antidysrhythmic medicine now because I have a pacemaker."

4."I can stop taking my antidysrhythmic medicine now because I have a pacemaker." Rationale:Clients with an ICD usually continue to receive antidysrhythmic medications after discharge from the hospital. The nurse should stress the importance of continuing to take these medications as prescribed. The nurse should provide clear instructions about the purposes of the medications, dosage schedule, and side effects or adverse effects to report. Clients should sit down if they feel an internal defibrillator shock. They cannot have an MRI because of the possible magnetic properties of the device. Also, knowledge of how to reach EMS is important.

The nurse is conducting a health history of a client with a primary diagnosis of heart failure. Which conditions reported by the client could play a role in exacerbating the heart failure? Select all that apply. 1.Emotional stress 2.Atrial fibrillation 3.Nutritional anemia 4.Peptic ulcer disease 5.Recent upper respiratory infection

1.Emotional stress 2.Atrial fibrillation 3.Nutritional anemia 5.Recent upper respiratory infection Rationale:Heart failure is precipitated or exacerbated by physical or emotional stress, dysrhythmias, infections, anemia, thyroid disorders, pregnancy, Paget's disease, nutritional deficiencies (thiamine, alcoholism), pulmonary disease, and hypervolemia. Peptic ulcer disease is not an exacerbating factor.

A client with no history of cardiovascular disease comes to the ambulatory clinic with flu-like symptoms. The client suddenly complains of chest pain. Which question should best help the nurse discriminate pain caused by a noncardiac problem? 1."Can you describe the pain to me?" 2."Have you ever had this pain before?" 3."Does the pain get worse when you breathe in?" 4."Can you rate the pain on a scale of 1 to 10, with 10 being the worst?"

3."Does the pain get worse when you breathe in?" Rationale:Chest pain is assessed by using the standard pain assessment parameters (e.g., characteristics, location, intensity, duration, precipitating and alleviating factors, and associated symptoms). The remaining options may or may not help discriminate the origin of pain. Pain of pleuropulmonary origin usually worsens on inspiration.

The nurse is assisting in the care of a client scheduled for cardioversion. The nurse plans to set the defibrillator to which starting energy range level, depending on the specific emergency health care provider (HCP) prescription? 1.120 joules 2.200 joules 3.250 joules 4.350 joules

1.120 joules Rationale:For cardioversion procedures, the defibrillator is charged to the energy level prescribed by the emergency HCP. Countershock usually is started at 120 to 200 joules. The number of joules in the remaining options are incorrect.

The nurse is educating the client about variant angina. Which statement by the client indicates that the teaching has been effective? 1."Variant angina is induced by exercise." 2."Variant angina occurs at the same time each day." 3."Variant angina occurs at lower levels of activity." 4."Variant angina is less predictable and a precursor of myocardial infarction."

2."Variant angina occurs at the same time each day." Rationale: Variant angina, or Prinzmetal's angina, is prolonged and severe and occurs at the same time each day, usually in the morning. Stable angina is induced by exercise and relieved by rest or nitroglycerin tablets. Unstable angina occurs at lower levels of activity than those that previously precipitated the angina. Unstable angina also occurs at rest, is less predictable, and is often a precursor of myocardial infarction.

The nurse is assessing the client's condition after cardioversion. Which observation should be of highest priority to the nurse? 1.Heart rate 2.Skin color 3.Status of airway 4.Peripheral pulse strength

3.Status of airway Rationale:Nursing responsibilities after cardioversion include maintenance of a patent airway, oxygen administration, assessment of vital signs and level of consciousness, and dysrhythmia detection. Airway, however, is always the highest priority,

The nurse is performing a health screening on a 54-year-old client. The client has a blood pressure of 118/78 mm Hg, total cholesterol level of 190 mg/dL (4.9 mmol/L), and fasting blood glucose level of 184 mg/dL (10.2 mmol/L). The nurse interprets this to mean that the client has which modifiable risk factor for coronary artery disease (CAD)? 1.Age 2.Hypertension 3.Hyperlipidemia 4.Glucose intolerance

4.Glucose intolerance Rationale:Hypertension, cigarette smoking, and hyperlipidemia are modifiable risk factors that are predictors of CAD. Glucose intolerance, obesity, and response to stress are contributing modifiable risk factors for CAD. Age is a nonmodifiable risk factor. The nurse places priority on risk factors that can be modified. In this scenario, the abnormal value is the fasting blood glucose level, indicating glucose intolerance as the priority risk factor.

The nurse is caring for a client with a diagnosis of myocardial infarction (MI) and is assisting the client in completing the diet menu. Which beverage should the nurse instruct the client to select from the menu? 1.Tea 2.Cola 3.Coffee 4.Raspberry juice

4.Raspberry juice Rationale:A client with a diagnosis of MI should not consume caffeinated beverages. Caffeinated products can produce a vasoconstrictive effect, leading to further cardiac ischemia. Coffee, tea, and cola all contain caffeine and need to be avoided in the client with MI.

The nurse is evaluating a client's response to cardioversion. Which assessment would be the priority? 1.Blood pressure 2.Status of airway 3.Oxygen flow rate 4.Level of consciousness

2.Status of airway Rationale:Nursing responsibilities after cardioversion include maintenance first of a patent airway and then oxygen administration, assessment of vital signs and level of consciousness, and dysrhythmia detection.

The nurse is providing instructions to a client with a diagnosis of hypertension regarding high-sodium items to be avoided. The nurse instructs the client to avoid consuming which item? 1.Bananas 2.Broccoli 3.Antacids 4.Cantaloupe

3.Antacids Rationale:The sodium level can increase with the use of several types of products, including toothpaste and mouthwash; over-the-counter medications such as analgesics, antacids, laxatives, and sedatives; and softened water and mineral water. Clients are instructed to read labels for sodium content. Water that is bottled, distilled, deionized, or demineralized may be used for drinking and cooking. Fresh fruits and vegetables are low in sodium.

A client with a history of hypertension has been prescribed triamterene. The nurse determines that the client understands the effect of this medication on the diet if the client states to avoid which fruit? 1.Pears 2.Apples 3.Bananas 4.Cranberries

3.Bananas Rationale:Triamterene is a potassium-retaining diuretic, so the client should avoid foods high in potassium. Fruits that are naturally higher in potassium include avocados, bananas, fresh oranges, mangos, nectarines, papayas, and prunes.

A client with myocardial infarction (MI) has been transferred from the coronary care unit (CCU) to the general medical unit. What activity level should the nurse encourage for the client immediately after transfer? 1.Ad lib activities as tolerated 2.Strict bed rest for 24 hours after transfer 3.Bathroom privileges and self-care activities 4.Unsupervised hallway ambulation for distances up to 200 feet (60 meters)

3.Bathroom privileges and self-care activities Rationale:On transfer from CCU to an intermediate care or general medical unit, the client is allowed self-care activities and bathroom privileges. Activities ad lib as tolerated is premature at this time and potentially harmful for this client. It is unnecessary and possibly harmful to limit the client to bed rest. The client should ambulate with supervision in the hall for brief distances, with the distances being gradually increased to 50, 100, and 200 feet (15, 30, and 60 meters).

A client has experienced an episode of pulmonary edema. The nurse determines that the client's respiratory status is improving after this episode if which breath sounds are noted? 1.Rhonchi 2.Wheezes 3.Crackles in the bases 4.Crackles throughout the lung fields

3.Crackles in the bases Rationale:Pulmonary edema is characterized by extreme breathlessness, dyspnea, air hunger, and the production of frothy, pink-tinged sputum. As the client's condition improves, the amount of fluid in the alveoli decreases, which may be detected by crackles in the bases. (Clear lung sounds indicate full resolution of the episode.) Rhonchi and wheezes are not associated with pulmonary edema. Auscultation of the lungs reveals crackles throughout the lung fields.

A client has developed uncontrolled atrial fibrillation with a ventricular rate of 150 beats/min. What manifestation should the nurse observe for when performing the client's focused assessment? 1.Flat neck veins 2.Nausea and vomiting 3.Hypotension and dizziness 4.Clubbed fingertips and headache

3.Hypotension and dizziness Rationale: The client with uncontrolled atrial fibrillation with a ventricular rate greater than 100 beats/min is at risk for low cardiac output due to loss of atrial kick. The nurse assesses the client for palpitations, chest pain or discomfort, hypotension, pulse deficit, fatigue, weakness, dizziness, syncope, shortness of breath, and distended neck veins.

The cardiologist has written a prescription for a client to have an echocardiogram. Which action should the nurse take to prepare the client for the procedure? 1.Questions the client about allergies to iodine or shellfish 2.Has the client sign an informed consent form for an invasive procedure 3.Tells the client that the procedure is painless and takes 30 to 60 minutes 4.Keeps the client on nothing by mouth (NPO) status for 2 hours before the procedure

3.Tells the client that the procedure is painless and takes 30 to 60 minutes Rationale:Echocardiography is a noninvasive, risk-free, pain-free test that involves no special preparation. It commonly is done at the bedside or on an outpatient basis. The client must lie quietly for 30 to 60 minutes while the procedure is being performed. It is important to provide adequate information to eliminate unnecessary worry on the part of the client.

The nurse has just completed education on myocardial infarction (MI) to a group of new nurses. Which statement made by one of the nurses indicates that the teaching has been effective? 1."Chest pain is caused by tissue hypoxia in the myocardium." 2."Chest pain is caused by tissue hypoxia in the vessels of the heart." 3."Chest pain is caused by tissue hypoxia in the parietal pericardium." 4."Chest pain is caused by tissue hypoxia in the visceral pericardium."

1."Chest pain is caused by tissue hypoxia in the myocardium." Rationale:The myocardial layer of the heart is damaged when a client experiences an MI. This is the middle layer that contains the striated muscle fibers responsible for the contractile force of the heart. The obstruction, which causes the interruption in blood flow and ensuing hypoxia, affects the myocardial layer. The endocardium is the thin inner layer of cardiac tissue. The parietal pericardium and visceral pericardium are outer layers that protect the heart from injury and infection.

An ambulatory care nurse measures the blood pressure of a client and finds it to be 156/94 mm Hg. Which statement indicates that the client needs additional education? 1."It is important that I limit protein intake." 2."I need to maintain a regular exercise program." 3."I understand that I need to avoid adding salt to foods." 4."It is important that I begin reducing and then maintaining weight."

1."It is important that I limit protein intake." Rationale:Obesity and sodium intake are modifiable risk factors for hypertension. These are of the utmost importance because they can be changed or modified by the individual through a regular exercise program and careful monitoring of sodium intake. Protein intake has no relationship to hypertension.

The nurse is listening to a lecture about angina. Which statement by the nurse indicates that the teaching has been effective? 1."Stable angina is chronic." 2."Variant angina is caused by emotional stress." 3."Unstable angina is not a life-threatening condition." 4."Intractable angina rarely limits the client's lifestyle."

1."Stable angina is chronic." Rationale:Stable angina is triggered by a predictable amount of effort or emotion and is a chronic condition. Variant angina is triggered by coronary artery spasm; the attacks are of longer duration than in classic angina and tend to occur early in the day and at rest. Unstable angina is triggered by an unpredictable amount of exertion or emotion and may occur at night; the attacks increase in number, duration, and severity over time. Intractable angina is chronic and incapacitating and is refractory to medical therapy.

A nursing student who is researching a medication at the nurses' station asks the registered nurse (RN) what the function of an alpha-adrenergic receptor is, and where the receptors are primarily found. The RN educates the nursing student. Which statement by the nursing student indicates that teaching has been effective? 1."The peripheral arteries and veins; when stimulated they cause vasoconstriction." 2."Arterial and bronchial walls; when stimulated they cause vasodilation and bronchodilation." 3."The heart; when stimulated it causes an increase in heart rate, atrioventricular node conduction, and contractility." 4."Several tissues; when stimulated they cause contraction of smooth muscle, inhibition of lipolysis, and promotion of platelet aggregation."

1."The peripheral arteries and veins; when stimulated they cause vasoconstriction." Rationale:Found in the peripheral arteries and veins, alpha-adrenergic receptors cause a powerful vasoconstriction when stimulated. The remaining options are incorrect statements.

A client with rapid-rate atrial fibrillation asks the nurse why the cardiologist is going to perform carotid sinus massage. The nurse educates the client about the treatment. Which statement by the client indicates that the teaching has been effective? 1."The vagus nerve slows the heart rate." 2."The diaphragmatic nerve slows the heart rate." 3."The diaphragmatic nerve overdrives the rhythm." 4."The vagus nerve increases the heart rate, overdriving the rhythm."

1."The vagus nerve slows the heart rate." Rationale:Carotid sinus massage is one maneuver used for vagal stimulation to decrease a rapid heart rate and possibly terminate a tachydysrhythmia. Others include inducing the gag reflex and asking the client to strain or bear down. Medication therapy is often needed as an adjunct to keep the rate down or maintain the normal rhythm. The remaining options are incorrect descriptions of this procedure.

The nurse is participating in a class on rhythm strip interpretation. Which statement by the nurse indicates an understanding of a PR interval of 0.20? 1. "This is a normal finding." 2. "This is indicative of atrial flutter." 3. "This is indicative of atrial fibrillation." 4. "This is indicative of impending reinfarction."

1."This is a normal finding." Rationale:The PR interval represents the time it takes for the cardiac impulse to spread from the atria to the ventricles. The normal range for the PR interval is 0.12 to 0.20 seconds. The remaining options are incorrect and indicate that further education is needed.

The nurse is assessing a client newly diagnosed with mild hypertension. Which assessment finding should the nurse expect? 1.Asymptomatic 2.Shortness of breath 3.Visual disturbances 4.Frequent nosebleeds

1.Asymptomatic Rationale:Hypertensive clients often have no symptoms until target organ involvement, which happens with very high blood pressure. This is why it is often noted as the "silent killer." The remaining options are incorrect because those clinical manifestations occur with severely high hypertension.

A client is scheduled for a cardiac catheterization to diagnose the extent of coronary artery disease. The nurse places highest priority on telling the client to report which sensation during the procedure? 1.Chest pain 2.Urge to cough 3.Warm, flushed feeling 4.Pressure at the insertion site

1.Chest pain Rationale:The client is taught to report chest pain or any unusual sensations immediately. The client also is told that he or she may be asked to cough or breathe deeply from time to time during the procedure. The client is informed that a warm, flushed feeling may accompany dye injection and is normal. Because a local anesthetic is used, the client is expected to feel pressure at the insertion site.

The nurse is assessing the neurovascular status of a client who returned to the surgical nursing unit 4 hours ago after undergoing aortoiliac bypass graft. The affected leg is warm, and the nurse notes redness and edema. The pedal pulse is palpable. How should the nurse interpret the client's neurovascular status? 1.The neurovascular status is normal because of increased blood flow through the leg. 2.The neurovascular status is moderately impaired, and the surgeon should be called. 3.The neurovascular status is slightly deteriorating and should be monitored for another hour. 4.The neurovascular status shows adequate arterial flow, but venous complications are arising.

1.The neurovascular status is normal because of increased blood flow through the leg. Rationale:An expected outcome of aortoiliac bypass graft surgery is warmth, redness, and edema in the surgical extremity because of increased blood flow. The remaining options are incorrect interpretations.

The nurse monitors the client for which condition as a complication of polycythemia vera? 1.Thrombosis 2.Hypotension 3.Cardiomyopathy 4.Pulmonary edema

1.Thrombosis Rationale: Polycythemia vera is a disorder of the bone marrow. It results in excessive production of white blood cells, red blood cells, and platelets. Clients with polycythemia vera are also more likely to form blood clots that can cause thrombi, strokes, myocardial infarctions, and abnormal bleeding. Clients with polycythemia vera are hypertensive; therefore, hypotension is incorrect. Cardiomyopathy and pulmonary edema are not concerns with this disorder.

The nurse is evaluating a client's cardiac rhythm strip to determine if there is proper function of the VVI mode pacemaker. Which denotes proper functioning? 1.Spikes precede all P waves and QRS complexes. 2.There are consistent spikes before each P wave. 3.Spikes occur before QRS complexes when intrinsic ventricular beats do not occur. 4.Spikes occur before all QRS complexes regardless of intrinsic ventricular activity.

3.Spikes occur before QRS complexes when intrinsic ventricular beats do not occur. Rationale:When a pacemaker is operating in the VVI mode, pacemaker spikes will be observed before the QRS complex if the client does not have his or her own intrinsic beat; therefore, options 1, 2, and 4 are incorrect.

A client's electrocardiogram shows that the ventricular rhythm is irregular and there are no discernible P waves. The nurse recognizes that this pattern is associated with which condition? 1.Atrial flutter 2.Atrial fibrillation 3.Third-degree atrioventricular (AV) block 4.First-degree AV block

2.Atrial fibrillation Rationale:With atrial fibrillation, the ventricular rhythm is irregular and there are usually no discernible P waves. Therefore, an atrial rhythm cannot be determined. In atrial flutter, the QRS complexes may be either regular or irregular, and the P waves occur as flutter waves. A client in third-degree AV block (also known as complete heart block) has regular atrial and ventricular rhythms, but there is no connection between the P waves and the QRS complexes. In other words, the PR interval is variable and the QRS complexes are normal or widened, with no relationship with the P waves. With first-degree AV block, the PR interval is longer than normal, and there is a connection between the occurrence of P waves and that of QRS complexes.

The nurse is concerned about the adequacy of peripheral tissue perfusion in the post-cardiac surgery client. Which action should the nurse include within the plan of care for this client? 1.Use the knee gatch on the bed. 2.Cover the legs lightly when sitting in a chair. 3.Encourage the client to cross the legs when sitting in a chair. 4.Provide pillows for the client to place under the knees as desired.

2.Cover the legs lightly when sitting in a chair. Rationale:Covering the legs with a light blanket during sitting promotes warmth and vasodilation of the leg vessels. The nurse plans postoperative measures to prevent venous stasis. These include applying elastic stockings or leg wraps, use of pneumatic compression boots, and discouraging crossing of the legs. Clients should be encouraged to perform passive and active range-of-motion exercises. The knee gatch on the bed and pillows under the knees should be avoided because they place pressure on the blood vessels in the popliteal area, impeding venous return.

The nurse is developing a plan of care for a client with varicose veins in whom skin breakdown occurred over the varicosities as a result of secondary infection. Which is a priority intervention? 1.Keep the legs aligned with the heart. 2.Elevate the legs higher than the heart. 3.Clean the skin with alcohol every hour. 4.Position the client onto the side during every shift.

2.Elevate the legs higher than the heart. Rationale:In the client with a venous disorder, the legs are elevated above the level of the heart to assist with the return of venous blood to the heart. Alcohol is very irritating and drying to tissues and should not be used in areas of skin breakdown. Option 4 specifies infrequent care intervals, so it is not the priority intervention.

The nurse is assessing a client with an abdominal aortic aneurysm. Which assessment finding by the nurse is unrelated to the aneurysm? 1.Pulsatile abdominal mass 2.Hyperactive bowel sounds in the area 3.Systolic bruit over the area of the mass 4.Subjective sensation of "heart beating" in the abdomen

2.Hyperactive bowel sounds in the area Rationale:Hyperactive bowel sounds are not related specifically to an abdominal aortic aneurysm. Not all clients with an abdominal aortic aneurysm exhibit symptoms. Those who do may describe a feeling of the "heart beating" in the abdomen when supine or being able to feel the mass throbbing. A pulsatile mass may be palpated in the middle and upper abdomen. A systolic bruit may be auscultated over the mass.

The nurse is caring for a client immediately after insertion of a permanent demand pacemaker via the right subclavian vein. Which activity will assist with preventing dislodgement of the pacing catheter? 1.Limiting both movement and abduction of the left arm 2.Limiting both movement and abduction of the right arm 3.Assisting the client to get out of bed and ambulate with a walker 4.Having the physical therapist do active range-of-motion exercises to the right arm

2.Limiting both movement and abduction of the right arm Rationale:In the first several hours after insertion of a permanent or temporary pacemaker, the most common complication is pacing electrode dislodgement. The nurse helps prevent this complication by limiting the client's activities of the arm on the side of the insertion site. Therefore, the remaining options are incorrect.

A client is admitted to the hospital with a diagnosis of pericarditis. The nurse should assess the client for which manifestation that differentiates pericarditis from other cardiopulmonary problems? 1.Anterior chest pain 2.Pericardial friction rub 3.Weakness and irritability 4.Chest pain that worsens on inspiration

2.Pericardial friction rub Rationale:A pericardial friction rub is heard when inflammation of the pericardial sac is present during the inflammatory phase of pericarditis. Anterior chest pain may be experienced with angina pectoris and myocardial infarction. Weakness and irritability are nonspecific complaints and could accompany a variety of disorders. Chest pain that worsens on inspiration is characteristic of both pericarditis and pleurisy.

The nurse reading the operative record of a client who had cardiac surgery notes that the client's cardiac output immediately after surgery was 3.2 L/min. Evaluation of the cardiac output results leads the nurse to make which conclusion? 1.The cardiac output is above the normal range. 2.The cardiac output is below the normal range. 3.The cardiac output is in the low-normal range. 4.The cardiac output is in the high-normal range.

2.The cardiac output is below the normal range. Rationale:The normal cardiac output for the adult can range from 4 to 7 L/min. Therefore, a cardiac output of 3.2 L/min is below normal range.

After instruction on the application of antiembolism stockings, the nurse determines that the client requires further teaching if which of these actions is performed? 1.The client puts on the stockings before getting out of bed. 2.The client bunches up the stockings for easier application. 3.The client ensures that stockings are pulled up all the way. 4.The client ensures that the rough seams of the stockings are on the outside.

2.The client bunches up the stockings for easier application. Rationale:When applying antiembolism stockings, the client should not bunch up the stockings. Instead, the client should place the hand inside the stocking and pull the heel out. The foot of the stocking should then be placed over the client's foot and the rest of the stocking pulled up the leg. This will help to prevent wrinkling and twisting of the stocking. The remaining options demonstrate correct application of the stockings.

The nurse is trying to determine the ability of the client with myocardial infarction (MI) to manage independently at home after discharge. Which statement by the client is the strongest indicator of the potential for difficulty after discharge? 1."I need to start exercising more to improve my health." 2."I will be sure to keep my appointment with the cardiologist." 3."I don't have anyone to help me with doing heavy housework at home." 4."I think I have a good understanding of what all my medications are for."

3."I don't have anyone to help me with doing heavy housework at home." Rationale:To ensure the best outcome, clients should be able to comply with instructions related to activity, diet, medications, and follow-up health care on discharge from the hospital after an MI. All of the options except the correct one indicate that the client will be successful in these areas.

A client with a diagnosis of angina pectoris is hospitalized for an angioplasty. The client returns to the nursing unit after the procedure, and the nurse provides instructions to the client regarding home care measures. Which statement, if made by the client, indicates an understanding of the instructions? 1."I need to cut down on cigarette smoking." 2."I am so relieved that my heart is repaired." 3."I need to adhere to my dietary restrictions." 4."I am so relieved that I can eat anything I want to now."

3."I need to adhere to my dietary restrictions." Rationale:After angioplasty, the client needs to be instructed on the specific dietary restrictions that must be followed. Making the recommended dietary and lifestyle changes will assist in preventing further atherosclerosis. Abrupt closure of the artery can occur if the dietary and lifestyle recommendations are not followed. Cigarette smoking needs to be stopped. An angioplasty does not repair the heart.

The home health nurse is visiting a client who has had a mechanical valve replacement for severe mitral valve stenosis. Which statement by the client reflects an understanding of specific postoperative care after this surgery? 1."I need to count my pulse every day." 2."I have to do deep-breathing exercises every 2 hours." 3."I need to throw away my straight razor and buy an electric razor." 4."I have to go to the bathroom frequently because of my medication."

3."I need to throw away my straight razor and buy an electric razor." Rationale:Mechanical valves require long-term anticoagulation to prevent clots from forming on the "foreign" object implanted in the client's body. Anticoagulation therapy requires clients to avoid any trauma or potential means of causing bleeding, such as the use of straight razors. Counting pulse, deep-breathing exercises, and going to the bathroom frequently are not specifically related to postoperative care after prosthetic mechanical valve replacement.

A client's total cholesterol level is 344 mg/dL (8.6 mmol/L), low-density lipoprotein cholesterol (LDL-C) level is 164 mg/dL (4.25 mmol/L), and high-density lipoprotein cholesterol (HDL-C) level is 30 mg/dL (1.2 mmol/L). Based on analysis of the data, how should the nurse direct client teaching? 1. The client should maintain the current dietary regimen but increase activity level. 2. Results are inconclusive unless the triglyceride level is also screened, so teaching is not indicated at this time. 3. The client is at high risk for cardiovascular disease, and measures to modify all identified risk factors should be taught. 4. The client is at low risk for cardiovascular disease, so the client should be encouraged to continue to follow the current regimen.

3.The client is at high risk for cardiovascular disease, and measures to modify all identified risk factors should be taught Rationale:In the absence of documented cardiovascular disease, the desired goal is to have the total cholesterol level lower than 200 mg/dL (<5 mmol/L). A desired LDL-C level for all individuals is lower than 100 mg/dL (<2.59 mmol/L), and a desirable HDL-C level is higher than 40 mg/dL (>1.55 mmol/L). Because the client's levels are outside the range to a significant degree for all three values, the client is at high risk for developing cardiovascular disease and requires teaching on risk factor reduction.

A client who has been exercising in a gymnasium stops to measure his pulse and places his fingers over both carotid arteries simultaneously. The nurse exercising nearby is correct when cautioning the client to check the pulse on only one side, primarily for which reason? 1.It is unnecessary to use both hands. 2.The client could occlude the trachea. 3.The heart rate and blood pressure could drop. 4.Feeling dual pulsations may lead to an incorrect measurement.

3.The heart rate and blood pressure could drop. Rationale:Applying pressure to both carotid arteries at the same time is contraindicated. Excess pressure to the baroreceptors in the carotid vessels could cause the heart rate and blood pressure to drop reflexively. In addition, the manual pressure could interfere with the flow of blood to the brain, causing possible dizziness and syncope. Although the information in the remaining options may be correct, these are not the primary reasons.

The nurse working in a long-term care facility is assessing a client who is experiencing chest pain. The nurse should interpret that the pain is most likely caused by myocardial infarction (MI) on the basis of what assessment finding? 1.The client is not experiencing dyspnea. 2.The client is not experiencing nausea or vomiting. 3.The pain has not been relieved by rest and nitroglycerin tablets. 4.The client says the pain began while she was trying to open a stuck dresser drawer.

3.The pain has not been relieved by rest and nitroglycerin tablets. Rationale:The pain of MI is not relieved by rest and nitroglycerin and requires opioid analgesics, such as morphine sulfate, for relief. The pain of angina may radiate to the left shoulder, arm, neck, or jaw. It often is precipitated by exertion or stress, is accompanied by few associated symptoms, and is relieved by rest and nitroglycerin. The pain of MI also may radiate to the left arm, shoulder, jaw, and neck. It typically begins spontaneously, lasts longer than 30 minutes, and frequently is accompanied by associated symptoms (i.e., nausea, vomiting, dyspnea, diaphoresis, or anxiety).

The nurse is caring for a client who is on strict bed rest and creates a plan of care with goals related to the prevention of deep vein thrombosis and pulmonary emboli. Which nursing action is most helpful in preventing these disorders from developing? 1.Restricting fluids 2.Placing a pillow under the knees 3.Encouraging active range-of-motion exercises 4.Applying a heating pad to the lower extremities

3.Encouraging active range-of-motion exercises Rationale:Clients at greatest risk for deep vein thrombosis and pulmonary emboli are immobilized clients. Basic preventive measures include early ambulation, leg elevation, active leg exercises, elastic stockings, and intermittent pneumatic calf compression. Keeping the client well hydrated is essential because dehydration predisposes to clotting. A pillow under the knees may cause venous stasis. Heat should not be applied without a primary health care provider's prescription.

A hospitalized client has been diagnosed with heart failure as a complication of hypertension. In explaining the disease process to the client, the nurse identifies which chamber of the heart as primarily responsible for the symptoms? 1.Left atrium 2.Right atrium 3.Left ventricle 4.Right ventricle

3.Left ventricle Rationale:Hypertension increases the workload of the left ventricle because the ventricle has to pump the stroke volume against increased resistance (afterload) in the major blood vessels. Over time this causes the left ventricle to fail, leading to signs and symptoms of heart failure. The remaining options are not the chambers that are primarily responsible for this disease process, although these chambers may be affected as the disease becomes more chronic.

The nurse has provided dietary instructions to a client with coronary artery disease. Which statement by the client indicates an understanding of the dietary instructions? 1."I'll need to become a strict vegetarian." 2."I should use polyunsaturated oils in my diet." 3."I need to substitute eggs and whole milk for meat." 4."I should eliminate all cholesterol and fat from my diet."

2."I should use polyunsaturated oils in my diet." Rationale: The client with coronary artery disease should avoid foods high in saturated fat and cholesterol such as eggs, whole milk, and red meat. These foods contribute to increases in low-density lipoproteins. The use of polyunsaturated oils is recommended to control hypercholesterolemia. It is not necessary to eliminate all cholesterol and fat from the diet. It is not necessary to become a strict vegetarian.

The nurse in the medical unit is reviewing the laboratory test results for a client who has been transferred from the intensive care unit (ICU). The nurse notes that a cardiac troponin T assay was performed while the client was in the ICU. The nurse determines that this test was performed to assist in diagnosing which condition? 1.Heart failure 2.Atrial fibrillation 3.Myocardial infarction 4.Ventricular tachycardia

3.Myocardial infarction Rationale:Cardiac troponin T or cardiac troponin I have been found to be a protein marker in the detection of myocardial infarction, and assay for this protein is used in some institutions to aid in the diagnosis of a myocardial infarction. The test is not used to diagnose heart failure, ventricular tachycardia, or atrial fibrillation.

The nurse is preparing to ambulate a client on the third day after cardiac surgery. What should the nurse plan to do to enable the client to best tolerate the ambulation? 1.Remove telemetry equipment. 2.Provide the client with a walker. 3.Premedicate the client with an analgesic. 4.Encourage the client to cough and breathe deeply.

3.Premedicate the client with an analgesic. Rationale:The nurse should encourage regular use of pain medication for the first 48 to 72 hours after cardiac surgery because analgesia will promote rest, decrease myocardial oxygen consumption resulting from pain, and allow better participation in activities such as coughing, deep breathing, and ambulation. Providing the client with a walker and encouraging the client to cough and breathe deeply will not help in tolerating ambulation. Removal of telemetry equipment is contraindicated unless prescribed.

The nurse is evaluating the condition of a client after pericardiocentesis performed to treat cardiac tamponade. Which observation would indicate that the procedure was effective? 1. Muffled heart sounds 2. A rise in blood pressure 3. Jugular venous distention 4. Client expressions of dyspnea

2.A rise in blood pressure Rationale:Following pericardiocentesis, the client usually expresses immediate relief. Heart sounds are no longer muffled or distant and blood pressure increases. Distended neck veins are a sign of increased venous pressure, which occurs with cardiac tamponade.

The nurse who is auscultating a 56-year-old client's apical heart rate before administering digoxin notes that the heart rate is 52 beats/min. The nurse should make which interpretation of this information? 1.Normal, because of the client's age 2.Abnormal, requiring further assessment 3.Normal, as a result of the effects of digoxin 4.Normal, because this is the reason the client is receiving digoxin

2.Abnormal, requiring further assessment Rationale:The normal heart rate is 60 to 100 beats/min in an adult. On auscultating a heart rate that is less than 60 beats/min, the nurse would not administer the digoxin and would report the finding to the primary health care provider for further instruction. The remaining options are incorrect interpretations because the heart rate of 52 beats/min is not normal.

The nurse is reinforcing instructions to a hospitalized client with heart block about the fundamental concepts regarding the cardiac rhythm. The nurse explains to the client that the normal site in the heart responsible for initiating electrical impulses is which site? 1.Bundle of His 2.Purkinje fibers 3.Sinoatrial (SA) node 4.Atrioventricular (AV) node

3.Sinoatrial (SA) node Rationale:The SA node is responsible for initiating electrical impulses that are conducted through the heart. The impulse leaves the SA node and travels down through internodal and interatrial pathways to the AV node. From there, impulses travel through the bundle of His to the right and left bundle branches and then to the Purkinje fibers. This group of specialized cardiac cells is referred to as the cardiac conduction system. The ability of this specialized tissue to generate its own impulses is called automaticity.

A client's electrocardiogram (ECG) strip shows atrial and ventricular rates of 70 complexes/minute. The PR interval is 0.16 second, the QRS complex measures 0.06 second, and the PP interval is slightly irregular. How should the nurse report this rhythm? 1.Sinus tachycardia 2.Sinus bradycardia 3.Sinus dysrhythmia 4.Normal sinus rhythm

3.Sinus dysrhythmia Rationale:Sinus dysrhythmia has all of the characteristics of normal sinus rhythm except for the presence of an irregular PP interval. This irregular rhythm occurs because of phasic changes in the rate of firing of the sinoatrial node, which may occur with vagal tone and with respiration. Cardiac output is not affected.

The home health nurse visits a client recovering after an episode of cardiogenic shock secondary to an anterior myocardial infarction (MI) and provides home care instructions to the client. Which statement by the client indicates an understanding of these home care measures? 1."I exercise every day after breakfast." 2."I've gained 8 lb (3.6 kg) since discharge." 3."I take an antacid when I experience epigastric pain." 4."I have planned periods of rest at 10:00 a.m. and 3:00 p.m. daily."

4."I have planned periods of rest at 10:00 a.m. and 3:00 p.m. daily." Rationale:The client recovering from an episode of cardiogenic shock secondary to an MI will require a progressive rehabilitation related to physical activity. The heart requires several months to heal from an uncomplicated MI. The complication of cardiogenic shock increases the recovery period for healing. Paced activities with planned rest periods will decrease the chance of experiencing angina or delayed healing. It is best to allow the meal to settle prior to activity in order to improve circulation to the heart during exercise. Epigastric pain or a weight gain of 8 lb (3.6 kg) is significant and should be reported to the primary health care provider, at which point follow-up should occur.

A client is scheduled to undergo cardiac catheterization for the first time, and the nurse provides instructions to the client. Which client statement indicates an understanding of the instructions? 1. "It will really hurt when the catheter is first put in." 2. "I will receive general anesthesia for the procedure." 3. "I will have to go to the operating room for this procedure." 4. "I probably will feel tired after the test from lying on a hard x-ray table for a few hours."

4."I probably will feel tired after the test from lying on a hard x-ray table for a few hours." Rationale:It is common for the client to feel fatigued after the cardiac catheterization procedure. A local anesthetic is used, so little to no pain is experienced with catheter insertion. General anesthesia is not used. Other preprocedure teaching points include the fact that the procedure is done in a darkened cardiac catheterization room. The x-ray table is hard and may be tilted periodically, and the procedure may take 1 to 2 hours. The client may feel various sensations with catheter passage and dye injection.

The nurse is discussing smoking cessation with a client diagnosed with coronary artery disease (CAD). Which statement should the nurse make to try to motivate the client to quit smoking? 1. "None of the cardiovascular effects are reversible, but quitting might prevent lung cancer." 2. "Because most of the damage has already been done, it will be all right to cut down a little at a time." 3. "If you totally quit smoking right now, you can cut your cardiovascular risk to zero within a year." 4. "If you quit now, your risk of cardiovascular disease will decrease to that of a nonsmoker in 3 to 4 years."

4."If you quit now, your risk of cardiovascular disease will decrease to that of a nonsmoker in 3 to 4 years." Rationale:The risks to the cardiovascular system from smoking are noncumulative and are not permanent. Three to 4 years after cessation, a client's cardiovascular risk is similar to that of a person who never smoked. In addition, tobacco use and passive smoking from "secondhand smoke" (also called environmental smoke) substantially reduce blood flow in the coronary arteries. The statements in the remaining options are incorrect.

A client with a diagnosis of varicose veins is scheduled for treatment by sclerotherapy and is receiving education about the procedure from the nurse. Which statement by the client indicates that the teaching has been effective? 1."It involves tying off the veins so that circulation is redirected in another area." 2."It involves surgically removing the varicosity, so anesthesia will be required." 3."It involves tying off the veins to prevent sluggishness of blood from occurring." 4."It involves injecting an agent into the vein to damage the vein wall and close it off."

4."It involves injecting an agent into the vein to damage the vein wall and close it off." Rationale:Sclerotherapy is the injection of a sclerosing agent into a varicosity. The agent damages the vessel and causes aseptic thrombosis, which results in vein closure. With no blood flow through the vessel, distention will not occur. The surgical procedure for varicose veins is vein ligation and stripping. This procedure involves tying off the varicose vein and large tributaries and then removing the vein with the use of a hook and wires applied through multiple small incisions in the leg. Other treatments include the application of radiofrequency (RF) energy, in which the vein is heated from the inside by the RF energy and shrinks; collateral veins nearby take over. Laser treatment is another alternative to surgery; in this treatment, a laser fiber is used to heat and close the main vessel that is contributing to the varicosity.

The nurse is listening to a lecture on Advanced Cardiac Life Support (ACLS). The instructor is discussing electrocardiographic (ECG) changes caused by myocardial ischemia. Which statement by the nurse indicates that teaching has been effective? 1."Tall, peaked T waves can indicate ischemia." 2."Prolonged PR interval can indicate ischemia." 3."Widened QRS complex can indicate ischemia." 4."ST segment elevation or depression can indicate ischemia."

4."ST segment elevation or depression can indicate ischemia." Rationale:An ECG taken during a chest pain episode captures ischemic changes, which include ST segment elevation or depression. Tall, peaked T waves may indicate hyperkalemia. A prolonged PR interval indicates first-degree heart block. A widened QRS complex indicates delay in intraventricular conduction, such as a bundle branch block.

Which is the priority assessment in the care of a client who is newly admitted to the hospital for acute arterial insufficiency of the left leg and moderate chronic arterial insufficiency of the right leg? 1.Monitor oxygen saturation with pulse oximetry. 2.Assess activity tolerance before and after exercise. 3.Observe the client's cardiac rhythm with telemetry. 4.Assess peripheral pulses with an ultrasonic Doppler device.

4.Assess peripheral pulses with an ultrasonic Doppler device. Rationale:Acute arterial insufficiency is associated with interruption of arterial blood flow to an organ, tissue, or extremity. It is associated with an acutely painful pasty-colored leg. The priority is for the nurse to perform a comprehensive assessment of peripheral circulation. When pulses are difficult to palpate, the Doppler device is useful to determine the presence of blood flow to the area. The Doppler directs sound waves toward the artery being examined, which emits an audible sound. The nurse must document that the pulse was present via Doppler and not palpation. Although the remaining options may be components of the assessment, they are not the priority.

A client with a complete heart block has had a permanent demand ventricular pacemaker inserted. The nurse assesses for proper pacemaker function by examining the electrocardiogram (ECG) strip for the presence of pacemaker spikes at what point? 1.Before each P wave 2.Just after each P wave 3.Just after each T wave 4.Before each QRS complex

4.Before each QRS complex Rationale:If a ventricular pacemaker is functioning properly, there will be a pacer spike followed by a QRS complex. An atrial pacemaker spike precedes a P wave if an atrial pacemaker is implanted.

The ambulatory care nurse is working with a client who has been diagnosed with Prinzmetal's (variant) angina. What should the nurse plan to teach the client about this type of angina? 1.It is most effectively managed by beta-blocking agents. 2.It has the same risk factors as stable and unstable angina. 3.It can be controlled with a low-sodium, high-potassium diet. 4.Generally it is treated with calcium channel-blocking agents.

4.Generally it is treated with calcium channel-blocking agents. Rationale:Prinzmetal's angina results from spasm of the coronary vessels and is treated with calcium channel blockers. Beta blockers are contraindicated because they may actually worsen the spasm. The risk factors are unknown, and this type of angina is relatively unresponsive to nitrates. Diet therapy is not specifically indicated.

The nurse is listening to a cardiologist explain the results of a cardiac catheterization to a client and family. The cardiologist tells the client that a blockage is present in the large blood vessel that supplies the anterior wall of the left ventricle. The nurse determines that the blockage is located in which area? 1.Circumflex coronary artery 2.Right coronary artery (RCA) 3.Posterior descending coronary artery (PDA) 4.Left anterior descending coronary artery (LAD)

4.Left anterior descending coronary artery (LAD) Rationale:The LAD bifurcates from the left main coronary artery to supply the anterior wall of the left ventricle and a few other structures. The circumflex coronary artery bifurcates from the left coronary artery and supplies the left atrium and the lateral wall of the left ventricle. The RCA supplies the right side of the heart, including the right atrium and right ventricle. The PDA supplies the posterior wall of the heart.

A client is being discharged from the hospital after being treated for infective endocarditis. The nurse should provide the client with which discharge instruction? 1.Take acetaminophen if the chest pain worsens. 2.Take antibiotics until the chest pain is fully resolved. 3.Use a firm-bristle toothbrush and floss vigorously to prevent cavities. 4.Notify all primary health care providers (PHCPs) of the history of infective endocarditis before any invasive procedures.

4.Notify all primary health care providers (PHCPs) of the history of infective endocarditis before any invasive procedures. Rationale:The client should alert any PHCP about the history of infective endocarditis before invasive dental, oral, or upper respiratory procedures. The PHCP should place the client on prophylactic antibiotics if one of these procedures is needed. Antibiotics should be taken for the full course of therapy. The client should notify the PHCP if chest pain worsens or if dyspnea or other symptoms occur. The client should use a soft toothbrush and floss carefully to avoid any trauma to the gums, which could provide a portal of entry for bacterial infection.

The nurse is performing an admission assessment on a client with a diagnosis of Raynaud's disease. How should the nurse assess for this disease? 1.Checking for a rash on the digits 2.Observing for softening of the nails or nail beds 3.Palpating for a rapid or irregular peripheral pulse 4.Palpating for diminished or absent peripheral pulses

4.Palpating for diminished or absent peripheral pulses Rationale:Raynaud's disease produces closure of the small arteries in the distal extremities in response to cold, vibration, or external stimuli. Palpation for diminished or absent peripheral pulses checks for interruption of circulation. Skin changes include hair loss, thinning or tightening of the skin, and delayed healing of cuts or injuries. The nails grow slowly, become brittle or deformed, and heal poorly around the nail beds when infected. Although palpation of peripheral pulses is correct, a rapid or irregular pulse would not be noted.

A client recovering from an exacerbation of left-sided heart failure is experiencing activity intolerance. Which change in vital signs during activity would be the best indicator that the client is tolerating mild exercise? 1.Oxygen saturation decreased from 96% to 91%. 2.Pulse rate increased from 80 to 104 beats per minute. 3.Blood pressure decreased from 140/86 to 112/72 mm Hg. 4.Respiratory rate increased from 16 to 19 breaths per minute.

4.Respiratory rate increased from 16 to 19 breaths per minute. Rationale:Vital signs that remain near baseline indicate good cardiac reserve with exercise. Only the respiratory rate remains within the normal range. In addition, it reflects a minimal increase. A pulse rate increase to a rate more than 100 beats per minute during mild exercise does not show tolerance, nor does a 5% decrease in oxygen saturation levels. In addition, blood pressure decreasing by more than 10 mm Hg is not a sign indicating tolerance of activity.

A client's electrocardiogram strip shows atrial and ventricular rates of 110 beats per minute. The PR interval is 0.14 seconds, the QRS complex measures 0.08 seconds, and the PP and RR intervals are regular. How should the nurse interpret this rhythm? 1.Sinus tachycardia 2.Sinus bradycardia 3.Sinus dysrhythmia 4.Normal sinus rhythm

1.Sinus tachycardia Rationale:Sinus tachycardia has the characteristics of normal sinus rhythm, including a regular PP interval and normal-width PR and QRS intervals; however, the rate is the differentiating factor. In sinus tachycardia, the atrial and ventricular rates are greater than 100 beats per minute.

A client's electrocardiogram strip shows atrial and ventricular rates of 110 beats/minute. The PR interval is 0.14 seconds, the QRS complex measures 0.08 seconds, and the PP and RR intervals are regular. How should the nurse correctly interpret this rhythm? 1.Sinus tachycardia 2.Sinus bradycardia 3.Sinus dysrhythmia 4.Normal sinus rhythm

1.Sinus tachycardia Rationale:Sinus tachycardia has the characteristics of normal sinus rhythm, including a regular PP interval and normal-width PR and QRS intervals; however, the rate is the differentiating factor. In sinus tachycardia, the atrial and ventricular rates are greater than 100 beats/minute.

The nurse is developing a plan of care for a client recovering from pulmonary edema. The nurse establishes a goal to have the client participate in activities that reduce cardiac workload. The nurse should identify which client action as contributing to this goal? 1.Using a bedside commode 2.Sleeping in the supine position 3.Elevating the legs when in bed 4.Using seasonings to improve the taste of food

1.Using a bedside commode Rationale:Using a bedside commode decreases the work of getting to the bathroom or struggling to use the bedpan. The supine position increases respiratory effort and decreases oxygenation. Elevating the client's legs increases venous return to the heart thus increasing cardiac workload. Seasonings may be high in sodium and promote further fluid retention.

A client with no history of heart disease has experienced acute myocardial infarction and has been given thrombolytic therapy with tissue plasminogen activator. What assessment finding should the nurse identify as an indicator that the client is experiencing complications of this therapy? 1.Tarry stools 2.Nausea and vomiting 3.Orange-colored urine 4.Decreased urine output

1.Tarry stools Rationale:Thrombolytic agents are used to dissolve existing thrombi, and the nurse should monitor the client for obvious or occult signs of bleeding. This includes assessment for obvious bleeding within the gastrointestinal (GI) tract, urinary system, and skin. It also includes Hematest testing of secretions for occult blood. The correct option is the only one that indicates the presence of blood.

The primary health care provider prescribes limited activity (bed rest and bathroom only) for a client who developed deep vein thrombosis (DVT) after surgery. What interventions should the nurse plan to include in the client's plan of care? Select all that apply. 1.Encourage coughing with deep breathing. 2.Place in high-Fowler's position for eating. 3.Encourage increased oral intake of water daily. 4.Place thigh-length elastic stockings on the client. 5.Place sequential compression boots on the client. 6.Encourage the intake of dark green, leafy vegetables.

1.Encourage coughing with deep breathing. 3.Encourage increased oral intake of water daily. 4.Place thigh-length elastic stockings on the client. Rationale:The client with DVT may require bed rest to prevent embolization of the thrombus resulting from skeletal muscle action, anticoagulation to prevent thrombus extension and allow for thrombus autodigestion, fluids for hemodilution and to decrease blood viscosity, and elastic stockings to reduce peripheral edema and promote venous return. While the client is on bed rest, the nurse prevents complications of immobility by encouraging coughing and deep breathing. Venous return is important to maintain because it is a contributing factor in DVT, so the nurse maintains venous return from the lower extremities by avoiding hip flexion, which occurs with high-Fowler's position. The nurse avoids providing foods rich in vitamin K, such as dark green, leafy vegetables, because this vitamin can interfere with anticoagulation, thereby increasing the risk of additional thrombi and emboli. The nurse also would not include use of sequential compression boots for an existing thrombus. They are used only to prevent DVT because they mimic skeletal muscle action and can disrupt an existing thrombus, leading to pulmonary embolism.

The nurse is developing a plan of care for a client who will be admitted to the hospital with a diagnosis of deep vein thrombosis (DVT) of the right leg. The nurse would include which intervention in the plan? 1.Maintain activity level as prescribed. 2.Maintain the affected leg in a dependent position. 3.Administer an opioid analgesic every 4 hours around the clock. 4.Apply cool packs to the affected leg for 20 minutes every 4 hours.

1.Maintain activity level as prescribed Rationale:Standard management for the client with DVT includes maintaining the activity level as prescribed by the PHCP; limb elevation; relief of discomfort with warm, moist heat; and analgesics as needed. Recent research is showing that ambulation, as previously thought, does not cause pulmonary embolism and does not cause the existing DVT to worsen. Therefore, the nurse should maintain the prescribed activity level. Opioid analgesics are not required to relieve pain, and pain normally is relieved with acetaminophen.

A client is having a follow-up primary health care provider (PHCP) office visit after vein ligation and stripping. The client describes a sensation of "pins and needles" in the affected leg. Which would be an appropriate action by the nurse based on evaluation of the client's comment? .Report the complaint to the PHCP. 2.Instruct the client to apply warm packs. 3.Reassure the client that this is only temporary. 4.Advise the client to take acetaminophen until it is gone.

1.Report the complaint to the PHCP. Rationale:Hypersensitivity or a sensation of pins and needles in the surgical limb may indicate temporary or permanent nerve injury following surgery. The saphenous vein and saphenous nerve run close together in the distal third of the leg. Although complications from this surgery can occur, they are relatively rare so this symptom should be reported. The actions in the remaining options are incorrect and could be harmful; in addition, they delay the possible need for intervention about the client's complaint. Although nerve damage can occur and is usually temporary and minimal and resolves within a few months, it is not appropriate to tell the client that this occurrence is only temporary. The complaint needs to be further assessed.

The nurse notes bilateral 2+ edema in the lower extremities of a client with myocardial infarction who was admitted 2 days ago. Based on this observation, what should the nurse plan to do first? 1.Review intake and output records for the last 2 days. 2.Prescribe daily weights starting on the following morning. 3.Request a sodium restriction of 1 g/day from the cardiologist. 4.Change the time of diuretic administration from morning to evening.

1.Review intake and output records for the last 2 days. Rationale:Edema, the accumulation of excess fluid in the interstitial spaces, can be measured by intake greater than output and by a sudden increase in weight. Therefore, the nurse should review intake and output records for the last 2 days. Diuretics should be given in the morning whenever possible to avoid nocturia. Strict sodium restrictions are reserved for clients with severe symptoms.

The registered nurse (RN) is orienting a new RN assigned to the care of a client with a cardiac disorder and is told that the client has an alteration in cardiac output. After educating the new RN about cardiac output, which statement made by the new RN indicates the need for further instruction? 1."A cardiac output of 2 L/min is normal." 2."A cardiac output of 4 L/min is normal." 3."A cardiac output of 6 L/min is normal." 4."A cardiac output of 7 L/min is normal."

1."A cardiac output of 2 L/min is normal." Rationale: The cardiac cycle consists of contraction and relaxation of the heart muscle. In adults, the cardiac output ranges from 4 to 7 L/min. Therefore, option 1 identifies a low cardiac output.

The registered nurse (RN) is educating a new nurse about aortic regurgitation. Which statement by the new nurse indicates that the teaching has been effective? 1."Failure of the aortic valve to close completely allows blood to flow retrograde through the aorta to the left ventricle." 2."Failure of the aortic valve to close completely allows blood to flow retrograde through the left ventricle to the left atrium." 3."Failure of the aortic valve to close completely allows blood to flow retrograde through the right ventricle to the right atrium." 4."Failure of the aortic valve to close completely allows blood to flow retrograde through the pulmonary artery to the right ventricle."

1."Failure of the aortic valve to close completely allows blood to flow retrograde through the aorta to the left ventricle." Rationale: The aortic valve separates the aorta from the left ventricle. The statements in the remaining options are inaccurate.

The nurse is caring for a client with cardiac disease who has been placed on a cardiac monitor. The nurse notes that the client has developed atrial fibrillation and has a rapid ventricular rate of 150 beats/minute. The nurse should next assess the client for which finding? 1.Hypotension 2.Flat neck veins 3.Complaints of nausea 4.Complaints of headache

1.Hypotension Rationale:The client with uncontrolled atrial fibrillation with a ventricular rate greater than 100 beats/minute is at risk for low cardiac output because of loss of atrial kick. The nurse assesses the client for palpitations, chest pain or discomfort, hypotension, pulse deficit, fatigue, weakness, dizziness, syncope, shortness of breath, and distended neck veins.

The home care nurse has taught a client with a problem of inadequate cardiac output about helpful lifestyle adaptations to promote health. Which statement by the client best demonstrates an understanding of the information provided? 1."I will eat enough daily fiber to prevent straining at stool." 2."I will try to exercise vigorously to strengthen my heart muscle." 3."I will drink 3000 to 3500 mL of fluid daily to promote good kidney function." 4."Drinking 2 to 3 oz of liquor each night will promote blood flow by enlarging blood vessels."

1."I will eat enough daily fiber to prevent straining at stool." Rationale:Standard home care instructions for a client with this problem include, among others, lifestyle changes such as decreased alcohol intake, avoiding activities that increase the demands on the heart, instituting a bowel regimen to prevent straining and constipation, and maintaining fluid and electrolyte balance. Consuming 3000 to 3500 mL of fluid and exercising vigorously will increase the cardiac workload.

The new registered nurse (RN) is orienting on the cardiac unit. Which statement by the new RN indicates an understanding of an early indication of fluid volume deficit due to blood loss? 1."Pulse rate will increase." 2."Blood pressure will decrease." 3."Edema will be present in the legs." 4."Crackles in the lungs will be present."

1."Pulse rate will increase." Rationale:The cardiac output is determined by the volume of the circulating blood, the pumping action of the heart, and the tone of the vascular bed. Early decreases in fluid volume are compensated for by an increase in the pulse rate. Although the blood pressure will decrease, it is not the earliest indicator. Edema and crackles in the lungs indicate an increase in fluid overload.

The nurse is performing an admission assessment on a client with a diagnosis of angina pectoris who takes nitroglycerin for chest pain at home. During the assessment the client complains of chest pain. The nurse should immediately ask the client which question? 1."Where is the pain located?" 2."Are you having any nausea?" 3."Are you allergic to any medications?" 4."Do you have your nitroglycerin with you?"

1."Where is the pain located?" Rationale:If a client complains of chest pain, the initial assessment question is to ask the client about the pain intensity, location, duration, and quality. Although the questions in the remaining options all may be components of the assessment, none of these questions is the initial assessment question for this client.

The nurse is assessing a client's legs for the presence of edema. The nurse notes that the client has mild pitting with slight indentation and no perceptible swelling of the leg. How should the nurse define and document this finding? 1.1+ edema 2.2+ edema 3.3+ edema 4.4+ edema

1.1+ edema Rationale:Edema is accumulation of fluid in the intercellular spaces and is not normally present. To check for edema, the nurse would imprint his or her thumbs firmly against the ankle malleolus or the tibia. Normally, the skin surface stays smooth. If the pressure leaves a dent in the skin, pitting edema is present. Its presence is graded on the following 4-point scale: 1+, mild pitting, slight indentation, no perceptible swelling of the leg; 2+, moderate pitting, indentation subsides rapidly; 3+, deep pitting, indentation remains for a short time, leg looks swollen; 4+, very deep pitting, indentation lasts a long time, leg is very swollen.

A client has been experiencing difficulty with completion of daily activities because of underlying cardiovascular disease, as evidenced by exertional fatigue and increased blood pressure. Which observation by the nurse best indicates client progress in meeting goals for this problem? 1.Ambulates 10 feet (3 meters) farther each day 2.Verbalizes the benefits of increasing activity 3.Chooses a healthy diet that meets caloric needs 4.Sleeps without awakening throughout the night

1.Ambulates 10 feet (3 meters) farther each day Rationale:Each of the options indicates a positive outcome on the part of the client. Both option 2 and the correct one relate to the client problem of difficulty with completion of daily activities. However, the question asks about progress. The correct option is more action-oriented and therefore is the better choice. Option 3 would most likely indicate progress if the client had a problem of inadequate nutritional intake. Option 4 would be a satisfactory outcome for a client experiencing difficulty sleeping.

A client is at risk for vasovagal attacks that cause bradydysrhythmias. The nurse would tell the client to avoid which actions to prevent this occurrence? Select all that apply. 1.Applying pressure on the eyes 2.Raising the arms above the head 3.Taking stool softeners on a daily basis 4.Bearing down during a bowel movement 5.Simulating a gag reflex when brushing the teeth

1.Applying pressure on the eyes 2.Raising the arms above the head 4.Bearing down during a bowel movement 5.Simulating a gag reflex when brushing the teeth Rationale:Vasovagal attacks or syncope occurs when the client faints because the body overreacts to certain triggers. The vasovagal syncope trigger causes the heart rate and blood pressure to drop suddenly. That leads to reduced blood flow to the brain, causing the client to briefly lose consciousness. The client at risk should be taught to avoid actions that stimulate the vagus nerve. Actions to avoid include raising the arms above the head, applying pressure over the carotid artery, applying pressure over the eyes, stimulating a gag reflex when brushing the teeth or putting objects into the mouth, and bearing down or straining during a bowel movement. Taking stool softeners is an important measure to prevent the bearing down and straining during a bowel movement.

The nurse is providing postoperative care for a client who had a percutaneous insertion of an inferior vena cava filter and was on heparin therapy before surgery. The nurse should be most concerned about monitoring for which potential complications? 1. Bleeding and infection 2. Thrombosis and infection 3. Bleeding and wound dehiscence 4. Wound dehiscence and evisceration

1.Bleeding and infection Rationale: After inferior vena cava filter insertion, the nurse inspects the surgical site for bleeding and signs and symptoms of infection. Heparin therapy also predisposes the client to bleeding. Thrombosis is unlikely because the client is on heparin therapy. Wound dehiscence and evisceration are not concerns because no abdominal incision is made.

The nurse is performing a cardiovascular assessment on a client. Which parameter would the nurse assess to gain the best information about the client's left-sided heart function? 1.Breath sounds 2.Peripheral edema 3.Hepatojugular reflux 4.Jugular vein distention

1.Breath sounds Rationale:The client with heart failure may present with different signs and symptoms according to whether the right or the left side of the heart is failing. Peripheral edema, jugular vein distention, and hepatojugular reflux are all indicators of impaired right-sided heart function. Breath sounds are an accurate indicator of left-sided heart function.

The nurse assesses the sternotomy incision of a client on the third day after cardiac surgery. The incision shows some slight puffiness along the edges and is non-reddened, with no apparent drainage. The client's temperature is 99º F (37.2º C) orally. The white blood cell count is 7500 mm3 (7.5 × 109/L). How should the nurse interpret these findings? 1.Incision is slightly edematous but shows no active signs of infection. 2.Incision shows early signs of infection, although the temperature is nearly normal. 3.Incision shows no sign of infection, although the white blood cell count is elevated. 4.Incision shows early signs of infection, supported by an elevated white blood cell count.

1.Incision is slightly edematous but shows no active signs of infection. Rationale:Sternotomy incision sites are assessed for signs and symptoms of infection, such as redness, swelling, induration, and drainage. An elevated temperature and white blood cell count 3 to 4 days postoperatively usually indicate infection. Therefore, the option indicating that there is slight edema and no active signs of infection is correct.

The nurse is performing an assessment on a client with a diagnosis of left-sided heart failure. Which assessment component would elicit specific information regarding the client's left-sided heart function? 1.Listening to lung sounds 2.Palpating for organomegaly 3.Assessing for jugular vein distention 4.Assessing for peripheral and sacral edema

1.Listening to lung sounds Rationale: The client with heart failure may present with different symptoms, depending on whether the right or the left side of the heart is failing. Peripheral and sacral edema, jugular vein distention, and organomegaly all are manifestations of problems with right-sided heart function. Lung sounds constitute an accurate indicator of left-sided heart function.

The nurse is assessing the neurovascular status of a client who returned to the surgical nursing unit 4 hours ago after undergoing aortoiliac bypass graft. The affected leg is warm, and the nurse notes redness and edema. The pedal pulse is palpable and unchanged from admission. How should the nurse correctly interpret the client's neurovascular status? 1. The neurovascular status is normal because of increased blood flow through the leg. 2. The neurovascular status is moderately impaired, and the surgeon should be called. 3. The neurovascular status is slightly deteriorating and should be monitored for another hour. 4. The neurovascular status is adequate from an arterial approach, but venous complications are arising.

1.The neurovascular status is normal because of increased blood flow through the leg. Rationale:An expected outcome of aortoiliac bypass graft surgery is warmth, redness, and edema in the surgical extremity because of increased blood flow. The remaining options are incorrect interpretations.

A client is seen in the emergency department for complaints of chest pain that began 3 hours ago. The nurse should suspect myocardial injury or infarction if which laboratory value comes back elevated? 1.Troponin 2.Myoglobin 3.C-reactive protein 4.Creatine kinase (CK)

1.Troponin Rationale:Cardiac troponin elevations indicate myocardial injury or infarction. Although the remaining options may also rise, they are not definitive enough to draw a conclusive diagnosis.

A client who has had a myocardial infarction asks the nurse why she should not bear down or strain to ensure having a bowel movement. The nurse provides education to the client based on which physiological concept? 1.Vagus nerve stimulation causes a decrease in heart rate and cardiac contractility. 2.Vagus nerve stimulation causes an increase in heart rate and cardiac contractility. 3.Sympathetic nerve stimulation causes a decrease in heart rate and cardiac contractility. 4.Sympathetic nerve stimulation causes an increase in heart rate and cardiac contractility.

1.Vagus nerve stimulation causes a decrease in heart rate and cardiac contractility. Rationale: Bearing down as if straining to have a bowel movement can stimulate a vagal reflex. Stimulation of the vagus nerve causes a decrease in heart rate and cardiac contractility. Stimulation of the sympathetic nervous system has the opposite effect. These two branches of the autonomic nervous system oppose each other to maintain homeostasis.

A client recovering from pulmonary edema is preparing for discharge. What should the nurse plan to teach the client to do to manage or prevent recurrent symptoms after discharge? 1.Weigh self on a daily basis. 2.Sleep with the head of the bed flat. 3.Take a double dose of the diuretic if peripheral edema is noted. 4.Withhold prescribed digoxin if slight respiratory distress occurs.

1.Weigh self on a daily basis. Rationale:The client can best determine fluid status at home by weighing himself or herself on a daily basis. Increases of 2 to 3 lb (0.9 to 1.4 kg) in a short period are reported to the primary health care provider (PHCP). The client should sleep with the head of the bed elevated. During recumbent sleep, fluid (which has seeped into the interstitium with the assistance of the effects of gravity) is rapidly reabsorbed into the systemic circulation. Sleeping with the head of the bed flat is therefore avoided. The client does not modify medication dosages without consulting the PHCP.

A client seeks treatment in a vascular surgeon's office for unsightly varicose veins, and radiofrequency ablation (RFA) is recommended. Before leaving the examining room, the client says to the nurse, "Can you tell me again how this is done?" Which statement should the nurse make? 1."The varicosity is surgically removed." 2."A heating element is used to occlude the vein." 3."The vein is tied off at the upper end to prevent stasis from occurring." 4."The vein is tied off at the lower end to prevent stasis from occurring."

2."A heating element is used to occlude the vein." Rationale:Radiofrequency ablation (RFA) is a treatment for varicose veins that uses a radiofrequency heat element to ablate (occlude) the affected vessel. This procedure is less invasive than a ligation and removal of veins procedure. Using ultrasound guidance, the clinician advances a catheter into a vein and injects an anesthetic agent around it. Then the vessel is ablated while the catheter is slowly removed. This causes collapse and sclerosis of the vein causing the occlusion.

The nurse has provided self-care activity instructions to a client after insertion of an implanted cardioverter-defibrillator (ICD). The nurse determines that further instruction is needed if the client makes which statement? 1."I need to avoid doing anything that could involve rough contact with the ICD insertion site." 2."I can perform activities such as swimming, driving, or operating heavy equipment as I need to." 3."I should try to avoid doing strenuous things that would make my heart rate go up to or above the rate cutoff on the ICD." 4."I should keep away from electromagnetic sources such as transformers, large electrical generators, and metal detectors, and I shouldn't lean over running motors."

2."I can perform activities such as swimming, driving, or operating heavy equipment as I need to." Rationale: Postdischarge instructions typically include avoiding tight clothing or belts over the ICD insertion sites; rough contact with the ICD insertion site; and electromagnetic fields such as with electrical transformers, radio/TV/radar transmitters, metal detectors, and running motors of cars or boats. Clients also must alert primary health care providers (PHCPs) or dentists to the presence of the device because certain procedures such as diathermy, electrocautery, and magnetic resonance imaging may need to be avoided to prevent device malfunction. Clients should follow the specific advice of a PHCP regarding activities that are potentially hazardous to self or others, such as swimming, driving, or operating heavy equipment.

The nurse is teaching the client with angina pectoris about disease management and lifestyle changes that are necessary to control disease progression. Which statement by the client indicates a need for further teaching? 1."I will avoid using table salt with meals." 2."It is best to exercise once a week for 1 hour." 3."I will take nitroglycerin whenever chest discomfort begins." 4."I will use muscle relaxation to cope with stressful situations."

2."It is best to exercise once a week for 1 hour." Rationale:Exercise is most effective when done at least 3 times a week for 20 to 30 minutes to reach a target heart rate. Other healthful habits include limiting salt and fat in the diet and using stress management techniques. The client also should be taught to take nitroglycerin before any activity that previously caused the pain and to take the medication at the first sign of chest discomfort.

The registered nurse (RN) is educating a new nurse on mitral stenosis. Which statement by the new nurse indicates that the teaching has been effective? 1."Left ventricle to aorta narrowing will impede flow of blood." 2."Left atrium to left ventricle narrowing will impede flow of blood." 3."Right atrium to right ventricle narrowing will impede flow of blood." 4."Right ventricle to pulmonary artery narrowing will impede flow of blood."

2."Left atrium to left ventricle narrowing will impede flow of blood." Rationale: The mitral valve separates the left atrium from the left ventricle. The remaining options describe impeded flow due to aortic, tricuspid, and pulmonic stenosis, respectively.

The nurse educator is lecturing new registered nurses (RNs) about serum calcium levels. Which statement by one of the new RNs indicates that teaching has been effective? 1."Calcium has no effect on the risk for stroke." 2."Low calcium levels can lead to cardiac arrest." 3."Low calcium levels cause high blood pressure." 4."Calcium has no effect on urinary stone formation."

2."Low calcium levels can lead to cardiac arrest." Rationale:The normal calcium level is 9 to 10.5 mg/dL (2.25 to 2.75 mmol/L). A low calcium level could lead to severe ventricular dysrhythmias, prolonged QT interval, and ultimately cardiac arrest. Calcium is needed by the heart for contraction. Calcium ions move across cell membranes into cardiac cells during depolarization and move back during repolarization. Depolarization is responsible for cardiac contraction. Options 1 and 3 are unrelated to calcium levels. Elevated calcium levels can lead to urinary stone formation. The nurse would take action and contact the health care provider when a calcium level is abnormal.

The nurse determines that a client requires further teaching after permanent pacemaker insertion if which statement is made? "I'll need to call my cardiologist if I feel tired or dizzy." 2. "My pulse rate should be less than what my pacemaker is set at." 3. "I'll have to avoid carrying the grocery bags into the house for the next 6 weeks." 4. "It's safe to use my microwave as long it is properly grounded and well shielded."

2."My pulse rate should be less than what my pacemaker is set at." Rationale:The client should call the cardiologist if the pulse rate is less than what the pacemaker is set at because this could be a sign of pacemaker or battery failure. Option 2 indicates the client needs further teaching, whereas the remaining options are correct statements.

The nurse is reviewing the procedure for performance of an electrocardiogram (ECG). Which action by the nurse indicates understanding of the correct position for the V1 lead when performing a 12-lead electrocardiogram? 1."The lead should be placed on the fourth intercostal space left sternal border." 2."The lead should be placed on the fourth intercostal space right sternal border." 3."The lead should be placed on the fifth intercostal space left midaxillary line." 4."The lead should be placed on the fifth intercostal space left midclavicular line."

2."The lead should be placed on the fourth intercostal space right sternal border." Rationale:The correct location for the V1 lead is the fourth intercostal space right sternal border. Therefore, the locations in the remaining options are incorrect.

The new registered nurse (RN) is reviewing cardiac rhythms with a mentor. Which statement by the new RN indicates that teaching about ventricular fibrillation has been effective? 1."Ventricular fibrillation appears as irregular beats within a rhythm." 2."Ventricular fibrillation does not have P waves or QRS complexes." 3."Ventricular fibrillation is a regular pattern of wide QRS complexes." 4."Ventricular fibrillation has recognizable P waves, QRS complexes, and T waves."

2."Ventricular fibrillation does not have P waves or QRS complexes." Rationale:Ventricular fibrillation is characterized by the absence of P waves and QRS complexes. The rhythm is instantly recognizable by the presence of coarse or fine fibrillatory waves on the cardiac monitoring screen. Premature ventricular contractions (PVCs) appear as irregular beats within a rhythm. Ventricular tachycardia is a regular pattern of wide QRS complexes. Sinus tachycardia has a recognizable P wave, QRS complex, and T wave. Each of the incorrect options has a recognizable complex that appears on the monitoring screen.

A client who had cardiac surgery 24 hours ago has had a urine output averaging 20 mL/hour for 2 hours. The client received a single bolus of 500 mL of intravenous fluid. Urine output for the subsequent hour was 25 mL. Daily laboratory results indicate that the blood urea nitrogen level is 45 mg/dL (16 mmol/L) and the serum creatinine level is 2.2 mg/dL (194 mcmol/L). On the basis of these findings, the nurse would anticipate that the client is at risk for which problem? 1.Hypovolemia 2.Acute kidney injury 3.Glomerulonephritis 4.Urinary tract infection

2.Acute kidney injury Rationale:The client who undergoes cardiac surgery is at risk for renal injury from poor perfusion, hemolysis, low cardiac output, or vasopressor medication therapy. Renal injury is signaled by decreased urine output and increased blood urea nitrogen (BUN) and creatinine levels. Normal reference levels are BUN, 10 to 20 mg/dL (3.6 to 7.1 mmol/L), and creatinine, male, 0.6 to 1.2 mg/dL (53 to 106 mcmol/L) and female 0.5 to 1.1 mg/dL (44 to 97 mcmol/L). The client may need medications to increase renal perfusion and possibly could need peritoneal dialysis or hemodialysis. No data in the question indicate the presence of hypovolemia, glomerulonephritis, or urinary tract infection.

The nurse is evaluating a client's response to cardioversion. Which assessment would be the priority? 1.Blood pressure 2.Airway patency 3.Oxygen flow rate 4.Level of consciousness

2.Airway patency Rationale:Nursing responsibilities after cardioversion include maintenance first of a patent airway, and then oxygen administration, assessment of vital signs and level of consciousness, and dysrhythmia detection.

Endovenous laser treatment (EVLT) is done on a client with varicose veins. Which interventions should the nurse include in the postprocedure plan of care? 1.Inform the client that the EVLT procedure ensures closure of the treated vein. 2.Assess color and temperature of the affected limb to determine vascular status. 3.Teach the client the importance of using graduated compression stockings (GCSs) during the day. 4.Inform the client that circulation impairment and nerve damage is expected to occur following the procedure.

2.Assess color and temperature of the affected limb to determine vascular status. Rationale:Endovenous laser treatment (EVLT) is a treatment for varicose veins that uses laser heat to ablate (occlude) the affected vessel. This procedure is less invasive than a ligation and removal of veins procedure. Using ultrasound guidance, the clinician advances a catheter into a vein (most commonly the saphenous vein) and injects an anesthetic agent around it. Then the vessel is ablated while the catheter is slowly removed. After the procedure, the client is taught the importance of using a GSC or other form of compression such as elastic compression bandages as prescribed for 24 hours a day (not just during the day), except for showers for at least the first week. A follow-up ultrasonography is done to ensure closure, so it is not appropriate to tell the client that the EVLT ensures closure; this needs to be verified. Circulation impairment is not expected. Nerve damage is not expected but can occur; if it does occur, it is usually temporary and minimal and resolves within a few months. The nurse needs to assess the vascular status of the affected limb and check for changes in color or temperature of the limb. The nurse would also monitor for pain, edema, and paresthesias that could indicate complications such as deep vein thrombosis or nerve damage.

The nurse is planning to teach a client with peripheral arterial disease about measures to limit disease progression. Which items should the nurse include on a list of suggestions for the client? Select all that apply. 1.Soak the feet in hot water daily. 2.Be careful not to injure the legs or feet. 3.Use a heating pad on the legs to aid vasodilation. 4.Walk each day to increase circulation to the legs. 5.Cut down on the amount of fats consumed in the diet.

2.Be careful not to injure the legs or feet. 4.Walk each day to increase circulation to the legs. 5.Cut down on the amount of fats consumed in the diet. Rationale:Long-term management of peripheral arterial disease consists of measures that increase peripheral circulation (exercise), promote vasodilation (warmth), relieve pain, and maintain tissue integrity (foot care and nutrition). Soaking the feet in hot water and application of a heating pad to the extremity are contraindicated. The affected extremity may have decreased sensitivity and is at risk for burns. Also, the affected tissue does not obtain adequate circulation at rest. Direct application of heat raises oxygen and nutritional requirements of the tissue even further.

The nurse is planning care for a client with deep vein thrombosis of the right leg. Which interventions would the nurse anticipate the physician to most likely prescribe? Select all that apply. 1.Strict bedrest 2.Elevation of the right leg 3.Administration of acetaminophen 4.Application of moist heat to the right leg 5.Monitoring for signs of pulmonary embolism

2.Elevation of the right leg 3.Administration of acetaminophen 4.Application of moist heat to the right leg 5.Monitoring for signs of pulmonary embolism Rationale: Standard management for the client with DVT includes maintaining the activity level as prescribed by the physician; limb elevation; relief of discomfort with warm, moist heat; and analgesics as needed. Strict bedrest is not likely to be prescribed; recent research is showing that ambulation does not cause pulmonary embolism and does not cause the existing DVT to worsen. Additionally, bedrest can cause complications such as skin integrity problems, weakness due to immobility, and respiratory problems.

A client complains of calf tenderness, and thrombophlebitis is suspected. The nurse should next assess the client for which finding? 1.Bilateral edema 2.Increased calf circumference 3.Diminished distal peripheral pulses 4.Coolness and pallor of the affected limb

2.Increased calf circumference Rationale:The client with thrombophlebitis, also known as deep vein thrombosis, exhibits redness or warmth of the affected leg, tenderness at the site, possibly dilated veins (if superficial), low-grade fever, edema distal to the obstruction, and increased calf circumference in the affected extremity. Peripheral pulses are unchanged from baseline because this is a venous, not an arterial, problem. Often thrombophlebitis develops silently; that is, the client does not present with any signs and symptoms unless pulmonary embolism occurs as a complication.

A client admitted to the hospital with chest pain and a history of type 2 diabetes mellitus is scheduled for cardiac catheterization. Which medication would need to be withheld for 24 hours before the procedure and for 48 hours after the procedure? 1.Glipizide 2.Metformin 3.Repaglinide 4.Regular insulin

2.Metformin Rationale: Metformin needs to be withheld 24 hours before and for 48 hours after cardiac catheterization because of the injection of a contrast medium during the procedure. If the contrast medium affects kidney function, with metformin in the system the client would be at increased risk for lactic acidosis. The medications in the remaining options do not need to be withheld 24 hours before and 48 hours after cardiac catheterization.

Spironolactone is prescribed for a client with heart failure. In providing dietary instructions to the client, the nurse identifies the need to avoid foods that are high in which electrolyte? 1.Calcium 2.Potassium 3.Magnesium 4.Phosphorus

2.Potassium Rationale:Spironolactone is a potassium-retaining diuretic, and the client should avoid foods high in potassium. If the client does not avoid foods high in potassium, hyperkalemia could develop. The client does not need to avoid foods that contain calcium, magnesium, or phosphorus while taking this medication.

The post-myocardial infarction client is scheduled for a technetium-99m ventriculography (multigated acquisition [MUGA] scan). The nurse ensures that which item is in place before the procedure? 1.A urinary catheter 2.Signed informed consent 3.A central venous pressure (CVP) line 4.Notation of allergies to iodine or shellfish

2.Signed informed consent Rationale:MUGA is a radionuclide study used to detect myocardial infarction and decreased myocardial blood flow and to determine left ventricular function. A radioisotope is injected intravenously; therefore, a signed informed consent is necessary. A urinary catheter and CVP line are not required. The procedure does not use radiopaque dye; therefore, allergies to iodine and shellfish are not a concern.

A client is scheduled for elective cardioversion to treat chronic high-rate atrial fibrillation. Which finding indicates that further preparation is needed for the procedure? 1. The client's digoxin has been withheld for the last 48 hours. 2. The client is wearing a nasal cannula delivering oxygen at 2 L/min. 3. The defibrillator has the synchronizer turned on and is set at 120 joules (J). 4. The client has received an intravenous dose of a conscious sedation medication.

2.The client is wearing a nasal cannula delivering oxygen at 2 L/min. Rationale: During the procedure, any oxygen is removed temporarily because oxygen supports combustion, and a fire could result from electrical arcing. Digoxin may be withheld for up to 48 hours before cardioversion because it increases ventricular irritability and may cause ventricular dysrhythmias after the countershock. The defibrillator is switched to synchronizer mode to time the delivery of the electrical impulse to coincide with the QRS and avoid the T wave, which could cause ventricular fibrillation. Energy level typically is set at 120 to 200 J for a biphasic machine. The client typically receives a dose of an intravenous sedative or antianxiety agent.

The nurse is assigned the care of a client with a diagnosis of heart failure who is receiving intravenous doses of furosemide. The client is attached to cardiac telemetry, and the nurse is monitoring the client's cardiac status. The nurse notes that the client's cardiac rhythm has changed to this pattern. The nurse determines that the most likely cause of this cardiac rhythm in the client is which problem? Refer to figure. 1.Pacemaker dysfunction 2.The presence of hypokalemia 3.The effectiveness of the furosemide 4.An impending myocardial infarction (MI)

2.The presence of hypokalemia Rationale:This cardiac rhythm is normal sinus rhythm with unifocal premature ventricular complexes (PVCs). PVCs may be insignificant, or they may occur with myocardial ischemia or MI; heart failure; hypokalemia; hypomagnesemia; medications; stress; nicotine, caffeine, or alcohol intake; infection; trauma; or surgery. This client is receiving furosemide, a diuretic that causes the excretion of potassium. The most likely cause of the PVCs in this client is hypokalemia. Option 3 is an incorrect interpretation. The question presents no data indicating that this client has a pacemaker or has signs or symptoms of an impending MI.

The nurse employed in a cardiac unit determines that which client is the least likely to have an implanted cardioverter-defibrillator (ICD) inserted? 1.A client with syncopal episodes related to ventricular tachycardia 2.A client with ventricular dysrhythmias despite medication therapy 3.A client with an episode of cardiac arrest related to myocardial infarction 4.A client with 3 episodes of cardiac arrest unrelated to myocardial infarction

3.A client with an episode of cardiac arrest related to myocardial infarction Rationale:An ICD detects and delivers an electrical shock to terminate life-threatening episodes of ventricular tachycardia and ventricular fibrillation. This device is implanted in clients who are considered high risk, including those who have syncopal episodes related to ventricular tachycardia, those who are refractive to medication therapy, and those who have survived sudden cardiac death unrelated to myocardial infarction.

A client calls the nurse at the clinic and reports that ever since the vein ligation and stripping procedure was performed, she has been experiencing a sensation as though the affected leg is falling asleep. The nurse should make which response to the client? 1. "Apply warm packs to the leg." 2. "Keep the leg elevated as much as possible." 3. "Your primary health care provider needs to be contacted to report this problem." 4. "This normally occurs after surgery and will subside when the edema goes down."

3."Your primary health care provider needs to be contacted to report this problem." Rationale: A sensation of pins and needles or feeling as though the surgical limb is falling asleep may indicate temporary or permanent nerve damage after surgery. The saphenous vein and the saphenous nerve run close together, and damage to the nerve will produce paresthesias. The remaining options are inaccurate responses. An alternative to surgery is endovenous ablation of the saphenous vein. Ablation involves the insertion of a catheter that emits energy. This causes collapse and sclerosis of the vein. Potential complications include bruising, tightness along the vein, recanalization (reopening of the vein), and paresthesia. Endovenous ablation also may be done in combination with saphenofemoral ligation or phlebectomy. Transilluminated powdered phlebectomy involves the use of a powdered resector to destroy the varices and then removes the pieces via aspiration.

The nurse educator is teaching the new registered nurse (RN) how to care for clients with a decrease in blood pressure. Which statement by the new RN indicates the need for further instruction? 1."Decreased contractility occurs." 2."Decreased heart rate is not a side effect." 3."Decreased myocardial blood flow is not a concern." 4."Increased resistance to electrical stimulation often occurs."

3."Decreased myocardial blood flow is not a concern." Rationale:The primary effect of a decrease in blood pressure is reduced blood flow to the myocardium. This in turn decreases oxygenation of the cardiac tissue. Cardiac tissue is likely to become more excitable or irritable in the presence of hypoxia. Correspondingly, the heart rate is likely to increase, not decrease, in response to this change. The effects of tissue ischemia lead to decreased contractility over time.

The registered nurse (RN) is listening to a lecture on pulmonary edema. Which statement by the RN indicates that the teaching has been effective? 1."The client may have mild anxiety." 2."The client will not experience anxiety." 3."The client will experience extreme anxiety." 4."The client will only experience anxiety in a stressful environment."

3."The client will experience extreme anxiety." Rationale:Pulmonary edema causes the client to be extremely agitated and anxious. The client may complain of a sense of drowning, suffocation, or smothering. Therefore, the client will experience extreme anxiety.

The registered nurse (RN) is educating a new RN about the use of oxygen for clients with angina pectoris. Which statement by the new nurse indicates that the teaching has been effective? 1."Oxygen has a calming effect." 2."Oxygen will prevent the development of any thrombus." 3."The pain of angina pectoris occurs because of a decreased oxygen supply to heart cells." 4."Oxygen dilates the blood vessels so that they can supply more nutrients to the heart muscle."

3."The pain of angina pectoris occurs because of a decreased oxygen supply to heart cells." Rationale:The pain associated with angina results from ischemia of myocardial cells. The pain often is precipitated by activity that places more oxygen demand on heart muscle. Supplemental oxygen will help meet the added demands on the heart muscle. Oxygen does not dilate blood vessels or prevent thrombus formation and does not directly calm the client.

A home care nurse is visiting a client to provide follow-up evaluation and care of a leg ulcer. On removing the dressing from the leg ulcer, the nurse notes that the ulcer is pale and deep and that the surrounding tissue is cool to the touch. The nurse should document that these findings identify which type of ulcer? 1.A stage 1 ulcer 2.A vascular ulcer 3.An arterial ulcer 4.A venous stasis ulcer

3.An arterial ulcer Rationale:Arterial ulcers have a pale deep base and are surrounded by tissue that is cool with trophic changes such as dry skin and loss of hair. Arterial ulcers are caused by tissue ischemia from inadequate arterial supply of oxygen and nutrients. A stage 1 ulcer indicates a reddened area with an intact skin surface. A venous stasis ulcer (vascular) has a dark red base and is surrounded by brown skin with local edema. This type of ulcer is caused by the accumulation of waste products of metabolism that are not cleared, as a result of venous congestion.

The nurse in the medical unit is assigned to provide discharge teaching to a client with a diagnosis of angina pectoris. The nurse is discussing lifestyle changes that are needed to minimize the effects of the disease process. The client continually changes the subject during the teaching session. The nurse interprets that this client's behavior is most likely related to which problem? 1. Anxiety related to the need to make lifestyle changes 2. Boredom resulting from having already learned the material 3. An attempt to ignore or deny the need to make lifestyle changes 4. Lack of understanding of the material provided at the teaching session and embarrassment about asking questions

3.An attempt to ignore or deny the need to make lifestyle changes Rationale:Denial is a defense mechanism that allows the client to minimize a threat that may be manifested by refusal to discuss what has happened. Denial is a common early reaction associated with chest discomfort, angina, or myocardial infarction (MI). Anxiety usually is manifested by symptoms of sympathetic nervous system arousal. No data are provided in the question that would lead the nurse to interpret the client's behavior as boredom or as either understanding or not understanding the material provided at the teaching session.

The nurse notes that a client's cardiac rhythm shows absent P waves, no PR interval, and an irregular rhythm. How should the nurse interpret this rhythm? 1.Bradycardia 2.Tachycardia 3.Atrial fibrillation 4.Normal sinus rhythm (NSR)

3.Atrial fibrillation Rationale: In atrial fibrillation, the P waves are absent and replaced by fibrillatory waves. There is no PR interval, and the QRS duration usually is normal and constant and the rhythm is irregular. Bradycardia is a slowed heart rate, and tachycardia is a fast heart rate. In NSR, a P wave precedes each QRS complex, the rhythm is essentially regular, the PR interval is 0.12 to 0.20 second, and the QRS interval is 0.06 to 0.10 second.

A client has been admitted with left-sided heart failure. When planning care for the client, interventions should be focused on reduction of which specific problem associated with this type of heart failure? 1.Ascites 2.Pedal edema 3.Bilateral lung crackles 4.Jugular vein distention

3.Bilateral lung crackles Rationale:The client with heart failure may present with different symptoms, depending on whether the right side or the left side of the heart is failing. Adventitious breath sounds, such as crackles, are an indicator of decreased left-sided heart function. Peripheral edema, jugular vein distention, and ascites all can be present because of insufficiency of the pumping action of the right side of the heart.

The client who is beginning an exercise program asks the nurse why his heart "feels like it's pounding" when he is exercising vigorously. The nurse provides education to the client about increased cardiac response based on which physiological concept? 1. Pulse rate is not a reflection of cardiac response. 2. Cardiac index is the mechanism that allows blood to flow better. 3. Cardiac output is the body's attempt to meet metabolic demands. 4. Stroke volume is an artificial number used to determine the adequacy of cardiac output.

3.Cardiac output is the body's attempt to meet metabolic demands. Rationale: The client's symptoms are the direct result of the body's attempt to meet the metabolic demands generated during exercise. An adequate cardiac output is needed to maintain perfusion to the vital organs of the body. With exercise, these demands increase, and the heart must beat faster (increased heart rate) and harder (increased stroke volume) to meet them. Cardiac index is an artificial number used to determine the adequacy of the cardiac output for a given individual. It is calculated by adjusting the cardiac output for body surface area.

A chaotic small, irregular, disorganized cardiac pattern suddenly appears on a client's cardiac monitor. Which is the nurse's first action? 1.Call the cardiologist. 2.Check the blood pressure. 3.Check the client and the chest leads. 4.Initiate cardiopulmonary resuscitation (CPR).

3.Check the client and the chest leads. Rationale:This type of pattern on the cardiac monitor indicates either ventricular fibrillation or lead displacement. The first action of the nurse is always to check the client and the chest leads. If the client is nonresponsive and the leads are not the problem, CPR would be the next choice, along with designating another person to contact the cardiologist.

The nurse is reviewing an electrocardiogram rhythm strip. The P waves and QRS complexes are regular. The PR interval is 0.16 seconds, and QRS complexes measure 0.06 seconds. The overall heart rate is 64 beats per minute. Which action should the nurse take? 1.Check vital signs. 2.Check laboratory test results. 3.Monitor for any rhythm change. 4.Notify the primary health care provider.

3.Monitor for any rhythm change. Rationale:Normal sinus rhythm is defined as a regular rhythm, with an overall rate of 60 to 100 beats per minute. The PR and QRS measurements are normal, measuring between 0.12 and 0.20 seconds and 0.04 and 0.10 seconds, respectively. There are no irregularities in this rhythm currently, so there is no immediate need to check vital signs or laboratory results, or to notify the primary health care provider. Therefore, the nurse would continue to monitor the client for any rhythm change.

The nurse is monitoring a client with acute pericarditis for signs of cardiac tamponade. Which assessment finding indicates the presence of this complication? 1.Flat neck veins 2.A pulse rate of 60 beats/minute 3.Muffled or distant heart sounds 4.Wheezing on auscultation of the lungs

3.Muffled or distant heart sounds Rationale:Assessment findings associated with cardiac tamponade include tachycardia, distant or muffled heart sounds, jugular vein distention with clear lung sounds, and a falling blood pressure accompanied by pulsus paradoxus (a drop in inspiratory blood pressure greater than 10 mm Hg). The other options are not signs of cardiac tamponade.

Which laboratory test results may be associated with peaked or tall, tented T waves on a client's electrocardiogram (ECG)? 1.Chloride level of 98 mEq/L (98 mmol/L) 2.Sodium level of 135 mEq/L (135 mmol/L) 3.Potassium level of 6.8 mEq/L (6.8 mmol/L) 4.Magnesium level of 1.6 mEq/L (0.8 mmol/L)

3.Potassium level of 6.8 mEq/L (6.8 mmol/L) Rationale: Hyperkalemia can cause tall, peaked, or tented T waves on the ECG. Potassium levels of 5.0 mEq/L (5.0 mmol/L) or greater indicate hyperkalemia. Options 1, 2, and 4 are normal levels.

The nurse is preparing discharge instructions for a client with Raynaud's disease. The nurse should plan to provide which instruction to the client? 1.Use nail polish to protect the nail beds from injury. 2.Wear gloves for all activities involving the use of both hands. 3.Stop smoking because it causes cutaneous blood vessel spasm. 4.Always wear warm clothing, even in warm climates, to prevent vasoconstriction.

3.Stop smoking because it causes cutaneous blood vessel spasm. Rationale:Raynaud's disease is peripheral vascular disease characterized by abnormal vasoconstriction in the extremities. Smoking cessation is one of the most important lifestyle changes that the client must make. The nurse should emphasize the effects of tobacco on the blood vessels and the principles involved in stopping smoking. The nurse needs to provide information to the client about smoking cessation programs available in the community. It is not necessary to wear gloves for all activities, nor should warm clothing be worn in warm climates.

The nurse is instructing the post-cardiac surgery client about activity limitations for the first 6 weeks after hospital discharge. The nurse should include which item in the instructions? 1.Driving is permitted as long as the lap and shoulder seat belts are worn. 2.Lifting should be restricted to objects that do not weigh more than 25 lb (11.3 kg). 3.Use the arms for balance, not weight support, when getting out of bed or a chair. 4.Activities that involve straining may be resumed as long as they do not cause pain.

3.Use the arms for balance, not weight support, when getting out of bed or a chair. Rationale: The client is taught to use the arms for balance, but not weight support, to avoid the effects of straining on the sternum. Typical discharge activity instructions for the first 6 weeks include instructing the client to lift nothing heavier than 5 lb (2.2 kg), to not drive, and to avoid any activities that cause straining. These limitations allow for sternal healing, which takes approximately 6 weeks.

An emergency room nurse is performing a cardiovascular assessment on a client. During auscultation of the heart sounds, the nurse hears these abnormal sounds. The nurse suspects that the client has which condition? 1.Atrial fibrillation 2.Ventricular fibrillation 3.Ventricular hypertrophy 4.Left bundle branch block

3.Ventricular hypertrophy Rationale:The sound that the nurse hears is the fourth heart sound (S4). Diastolic filling sounds or gallops (S3, the third heart sound, and S4, the fourth heart sound) are produced when there is decreased compliance of either or both ventricles. S3 is termed ventricular gallop and S4 is referred to as atrial gallop. The S4 sound occurs in the later stage of diastole, during atrial contraction and active filling of the ventricles. It is a soft, low-pitched sound and is heard immediately before S1. An atrial gallop is found most commonly in disorders involving increased stiffness of the ventricle, such as ventricular hypertrophy, ischemia, and fibrosis. S4 is never heard in the absence of atrial contraction (atrial fibrillation). S4 is best heard with the bell of the stethoscope at the apex, with the client in the supine, left lateral position. The presence of S4 may also result from myocardial infarction, hypertension, hypertrophy, fibrosis, cardiomyopathy, cor pulmonale, aortic stenosis, or pulmonic stenosis. Therefore, options 1, 2, and 4 are incorrect.

A client is attached to a cardiac monitor, and the nurse notes the presence of this cardiac rhythm on the monitor. The nurse quickly assesses the client, knowing that this rhythm is indicative of which rhythm? Refer to figure. 1.Atrial fibrillation 2.Ventricular fibrillation (VF) 3.Ventricular tachycardia (VT) 4.Premature ventricular complexes

3.Ventricular tachycardia (VT) Rationale:In VT, it usually is not possible to determine the atrial rhythm. The ventricular rhythm usually is regular or nearly regular. The P waves usually are not visible and are obscured in the QRS complexes. VT occurs with repetitive firing of an irritable ventricular ectopic focus, usually at a rate of 140 to 180 beats/min or more.

A client with coronary artery disease is scheduled to have a diagnostic exercise stress test. Which instruction should the nurse plan to provide to the client about this procedure? 1.Eat breakfast just before the procedure. 2.Wear firm, rigid shoes, such as work boots. 3.Wear loose clothing with a shirt that buttons in front. 4.Avoid cigarettes for 30 minutes before the procedure.

3.Wear loose clothing with a shirt that buttons in front. Rationale: The client should wear loose, comfortable clothing for the procedure. Electrocardiogram (ECG) lead placement is enhanced if the client wears a shirt that buttons in the front. The client should receive nothing by mouth after bedtime or for a minimum of 2 hours before the test. The client should wear rubber-soled, supportive shoes, such as athletic training shoes. The client should avoid smoking, alcohol, and caffeine on the day of the test. Inadequate or incorrect preparation can interfere with the test, with the potential for a false-positive result.

A client with variant angina is scheduled to receive an oral calcium channel blocker twice daily. Which statement by the client indicates the need for further teaching? 1. "I should notify my doctor if my feet or legs start to swell." 2. "My doctor told me to call his office if my pulse rate decreases below 60." 3. "Avoiding grapefruit juice will definitely be a challenge for me, since I usually drink it every morning with breakfast." 4. "My spouse told me that since I have developed this problem, we are going to stop walking in the mall every morning."

4. "My spouse told me that since I have developed this problem, we are going to stop walking in the mall every morning." Rationale:Variant angina, or Prinzmetal's angina, is prolonged and severe and occurs at the same time each day, most often at rest. The pain is a result of coronary artery spasm. The treatment of choice is usually a calcium channel blocker, which relaxes and dilates the vascular smooth muscle, thus relieving the coronary artery spasm in variant angina. Adverse effects can include peripheral edema, hypotension, bradycardia, and heart failure. Grapefruit juice interacts with calcium channel blockers and should be avoided. If bradycardia occurs, the client should contact the health care provider. Clients should also be taught to change positions slowly to prevent orthostatic hypotension. Physical exertion does not cause this type of angina; therefore, the client should be able to continue morning walks with his or her spouse.

A client with myocardial infarction is experiencing new, multiform premature ventricular contractions and short runs of ventricular tachycardia. The nurse plans to have which medication available for immediate use to treat the ventricular tachycardia? 1.Digoxin 2.Verapamil 3.Acebutolol 4.Amiodarone

4.Amiodarone Rationale:Amiodarone is an antidysrhythmic that may be used to treat ventricular dysrhythmias. Digoxin is a cardiac glycoside; verapamil is a calcium channel-blocking agent; acebutolol is a beta-adrenergic blocking agent. Digoxin can be used to treat supraventricular dysrhythmias but is inactive against ventricular dysrhythmias. Verapamil is used to slow the ventricular rate for a client with atrial fibrillation or atrial flutter, or to terminate supraventricular tachycardia. Acebutolol is a beta blocker used to treat dysrhythmias.

A client is scheduled for a cardiac catheterization using an iodine agent. Which assessment is most critical before the procedure? 1.Intake and output 2.Height and weight 3.Baseline peripheral pulse rates 4.Previous allergy to contrast agents

4.Previous allergy to contrast agents Rationale:A cardiac catheterization requires an informed consent because it involves injection of an iodine-based contrast agent into the blood vessel. The risk of allergic reaction and possible anaphylaxis is a concern, and the presence of allergies to previously received contrast agents must be assessed before the procedure. An antihistamine or corticosteroid may be given to a client with a positive history or to prevent a reaction. Although the remaining options are accurate, they are not the most critical preprocedure assessments.

A client with iron deficiency anemia complains of feeling fatigued almost all of the time. The nurse should respond with which statement? 1."The work of breathing is increased when the client is anemic." 2."Blood flows more slowly when the hemoglobin or hematocrit is low." 3."The body has to work harder to fight infection in the presence of anemia." 4."Adequate amounts of hemoglobin are needed to carry oxygen for tissue metabolism."

4."Adequate amounts of hemoglobin are needed to carry oxygen for tissue metabolism." Rationale:Oxygen is required to meet the metabolic needs of the body. With decreased hemoglobin, such as in iron deficiency anemia, the oxygen-carrying capacity of the blood is less than normal. The client feels the effects of this change as fatigue. The statements in the remaining options are incorrect.

The home care nurse is providing instructions to a client with an arterial ischemic leg ulcer about home care management and self-care management. Which statement, if made by the client, indicates a need for further instruction? 1."I need to be sure not to go barefoot around the house." 2."If I cut my toenails, I need to be sure that I cut them straight across." 3."It is all right to apply lanolin to my feet, but I shouldn't place it between my toes." 4."I need to be sure that I elevate my leg above the level of my heart for at least an hour every day."

4."I need to be sure that I elevate my leg above the level of my heart for at least an hour every day." Rationale:Foot care instructions for the client with peripheral arterial disease are the same as those for a client with diabetes mellitus. The client with arterial disease, however, should avoid raising the legs above the level of the heart unless instructed to do so as part of an exercise program or if venous stasis is also present. The client statements in the remaining options are correct statements and indicate that the teaching has been effective.

A client who had coronary artery bypass surgery states to the home health nurse, "I get so frustrated. I can't even do my gardening." The nurse then assesses the client for activity level since the surgery. Which client statement indicates a need for further teaching? 1."I pace my activities throughout the day." 2."I plan regular rest periods during the day." 3."I avoid outdoor physical activity during the heat of the day." 4."I try to walk immediately after lunch, after I've finished my morning housecleaning."

4."I try to walk immediately after lunch, after I've finished my morning housecleaning." Rationale:Exercise is an integral part of the rehabilitation program. It is necessary for optimal physiological functioning and psychological well-being. Postoperative physical rehabilitation must be progressive, with planned periods of rest. Exercise tolerance is judged by the client's response, such as heart rate and endurance. Planning regular rest periods, pacing activities, and avoiding outdoor activities during the heat of the day are appropriate client activities. The correct option lacks planned periods of rest, and the client has grouped too many activities in a brief period of time, which will decrease endurance. Also, exercise after meals can decrease the client's tolerance because of shunting of blood to the gastrointestinal tract for digestion.

A client is admitted to the visiting nurse service for assessment and follow-up after being discharged from the hospital with new-onset heart failure (HF). The nurse teaches the client about the dietary restrictions required with HF. Which statement by the client indicates that further teaching is needed? 1."I'm not supposed to eat cold cuts." 2."I can have most fresh fruits and vegetables." 3."I'm going to weigh myself daily to be sure I don't gain too much fluid." 4."I'm going to have a ham and cheese sandwich and potato chips for lunch."

4."I'm going to have a ham and cheese sandwich and potato chips for lunch." Rationale: When a client has HF, the goal is to reduce fluid accumulation. One way that this is accomplished is through sodium reduction. Ham (and most cold cuts), cheese, and potato chips are high in sodium. Daily weighing is an appropriate intervention to help the client monitor fluid overload. Most fresh fruits and vegetables are low in sodium.

The nurse has completed an educational course covering first-degree heart block. Which statement by the nurse indicates that teaching has been effective? 1."Presence of Q waves indicates first-degree heart block." 2."Tall, peaked T waves indicate first-degree heart block." 3."Widened QRS complexes indicate first-degree heart block." 4."Prolonged, equal PR intervals indicate first-degree heart block."

4."Prolonged, equal PR intervals indicate first-degree heart block." Rationale:Prolonged and equal PR intervals indicate first-degree heart block. The development of Q waves indicates myocardial necrosis. Tall, peaked T waves may indicate hyperkalemia. A widened QRS complex indicates a delay in intraventricular conduction, such as bundle branch block. An electrocardiogram (ECG) taken during a pain episode is intended to capture ischemic changes, which also include ST segment elevation or depression.

A nursing instructor asks a nursing student to describe the structure and function of the coronary arteries. Which response by the student indicates a need for further teaching on the anatomy and physiology of the heart? 1."The coronary arteries branch from the aorta." 2."The coronary arteries supply the heart muscle with blood." 3."The left coronary artery provides blood for the left atrium and the left ventricle." 4."The left coronary artery supplies the right atrium and right ventricle with blood."

4."The left coronary artery supplies the right atrium and right ventricle with blood." Rationale: The left coronary artery divides into the anterior descending artery and the circumflex artery, providing blood for the left atrium and left ventricle. The right coronary artery supplies the right atrium and right ventricle. Options 1, 2, and 3 are correct.

The nurse is caring for a client who has just had implantation of an automatic internal cardioverter-defibrillator. The nurse should assess which item based on priority? 1.Anxiety level of the client and family 2.Presence of a MedicAlert card for the client to carry 3.Knowledge of restrictions on postdischarge physical activity 4.Activation status of the device, heart rate cutoff, and number of shocks it is programmed to deliver

4.Activation status of the device, heart rate cutoff, and number of shocks it is programmed to deliver Rationale:The nurse who is caring for the client after insertion of an automatic internal cardioverter-defibrillator needs to assess device settings, similar to after insertion of a permanent pacemaker. Specifically, the nurse needs to know whether the device is activated, the heart rate cutoff above which it will fire, and the number of shocks it is programmed to deliver. The remaining options are also nursing interventions but are not the priority.

The nurse is reviewing an electrocardiogram rhythm strip. The P waves and QRS complexes are regular. The PR interval is 0.16 seconds, and QRS complexes measure 0.06 seconds. The overall heart rate is 64 beats/minute. Which action should the nurse take? 1. Check vital signs. 2. Check laboratory test results. 3. Notify the primary health care provider. 4. Continue to monitor for any rhythm change.

4.Continue to monitor for any rhythm change. Rationale:Normal sinus rhythm is defined as a regular rhythm, with an overall rate of 60 to 100 beats/minute. The PR and QRS measurements are normal, measuring between 0.12 and 0.20 seconds and 0.04 and 0.10 seconds, respectively. There are no irregularities in this rhythm currently, so there is no immediate need to check vital signs or laboratory results, or to notify the primary health care provider. Therefore, the nurse would continue to monitor the client for any rhythm change.

A client with an acute respiratory infection is admitted to the hospital with a diagnosis of sinus tachycardia. Which nursing action should be included in the client's plan of care? 1.Limiting oral and intravenous fluids 2.Measuring the client's pulse each shift 3.Providing the client with short, frequent walks 4.Eliminating sources of caffeine from meal trays

4.Eliminating sources of caffeine from meal trays Rationale:Sinus tachycardia often is caused by fever, physical and emotional stress, heart failure, hypovolemia, certain medications, nicotine, caffeine, and exercise. Fluid restriction and exercise will not alleviate tachycardia. Measuring the pulse each shift will not decrease the heart rate. In addition, the pulse should be taken more frequently than each shift.

The nurse is giving discharge instructions to a client who has just undergone vein ligation and stripping. The nurse evaluates that the client understands activity and positioning limitations if the client states that which action is appropriate to do? 1.Walk for as long as possible each day. 2.Cross the legs at the ankle only, not at the knee. 3.Sit in a chair 3 times a day for 3 hours at a time. 4.Lie down with the legs elevated and avoid sitting.

4.Lie down with the legs elevated and avoid sitting. Rationale: The client who has had vein ligation and stripping should avoid standing or sitting for prolonged periods. The client should remain lying down unless performing a specific activity for the first few days after the procedure. Prolonged standing or sitting increases the risk of edema in the legs by decreasing blood return to the heart. The client should avoid crossing the legs at any level for the same reason.

The nurse is teaching a client with cardiomyopathy about home care safety measures. The nurse should address with the client which most important measure to ensure client safety? 1.Assessing pain 2.Administering vasodilators 3.Avoiding over-the-counter (OTC) medications 4.Moving slowly from a sitting to a standing position

4.Moving slowly from a sitting to a standing position Rationale:Orthostatic changes can occur in the client with cardiomyopathy as a result of venous return obstruction. Sudden changes in blood pressure may lead to falls. Vasodilators normally are not prescribed for the client with cardiomyopathy. Although important, pain assessment and avoiding OTC medications are not directly related to the issue of safety.

A client admitted to the hospital with coronary artery disease complains of dyspnea at rest. The nurse caring for the client uses which item as the best means to monitor respiratory status on an ongoing basis? 1.Apnea monitor 2.Oxygen flowmeter 3.Telemetry cardiac monitor 4.Oxygen saturation monitor

4.Oxygen saturation monitor Rationale:Dyspnea in the cardiac client often is accompanied by hypoxemia. Hypoxemia can be detected by an oxygen saturation monitor, especially if it is used continuously. An apnea monitor detects apnea episodes, such as when the client has stopped breathing briefly. An oxygen flowmeter is part of the setup for delivering oxygen therapy. Cardiac monitors detect dysrhythmias.

A client with pulmonary edema has been receiving diuretic therapy. The client has a prescription for additional furosemide in the amount of 40 mg intravenous push. Knowing that the client will also be started on digoxin, which laboratory result should the nurse review as the priority? 1.Sodium level 2.Digoxin level 3.Creatinine level 4.Potassium level

4.Potassium level Rationale:Diuretic therapy can cause hypokalemia. The serum potassium level is measured in the client receiving digoxin and furosemide. Heightened digoxin effect leading to digoxin toxicity can occur in the client with hypokalemia. Hypokalemia also predisposes the client to ventricular dysrhythmias.

Cardiac monitoring leads are placed on a client who is at risk for premature ventricular contractions (PVCs). Which heart rhythm will the nurse anticipate in this client if PVCs are occurring? 1.A P wave preceding every QRS complex 2.QRS complexes that are short and narrow 3.Inverted P waves before the QRS complexes 4.Premature beats followed by a compensatory pause

4.Premature beats followed by a compensatory pause Rationale:PVCs are abnormal ectopic beats originating in the ventricles. They are characterized by an absence of P waves, the presence of wide and bizarre QRS complexes, and a compensatory pause that follows the ectopy.

The nurse is caring for a postoperative client who has lost a significant amount of blood because of complications during a surgical procedure. Which assessment finding would be indicative of further fluid volume deficit? 1.4+ edema noted in lower extremities 2.Crackles auscultated from lung bases to apices 3.Blood pressure rises from 116/68 to 118/74 mm Hg 4.Pulse rate increases from 100 beats/min to 136 beats/min

4.Pulse rate increases from 100 beats/min to 136 beats/min Rationale: The cardiac output is determined by the volume of the circulating blood, the pumping action of the heart, and the tone of the vascular bed. An increase in the pulse rate compensates for decreases in fluid volume. Options 1 and 2 may be noted in fluid overload. A low blood pressure is expected in a postoperative client who lost a significant amount of blood.

The nurse identifies that a client is having occasional premature ventricular contractions (PVCs) on the cardiac monitor. The nurse reviews the client's laboratory results and determines that which result would be consistent with the observation? 1.Serum chloride level of 98 mEq/L (98 mmol/L) 2.Serum sodium level of 145 mEq/L (145 mmol/L) 3.Serum calcium level of 10.5 mg/dL (2.75 mmol/L) 4.Serum potassium level of 2.8 mEq/L (2.8 mmol/L)

4.Serum potassium level of 2.8 mEq/L (2.8 mmol/L) Rationale:The nurse should check the client's serum laboratory study results for hypokalemia. The client may experience PVCs in the presence of hypokalemia because this electrolyte imbalance increases the electrical instability of the heart. The values noted in the remaining options are normal.

A client has been diagnosed with thromboangiitis obliterans (Buerger's disease). The nurse is identifying measures to help the client cope with lifestyle changes needed to control the disease process. The nurse plans to refer the client to which member of the health care team? 1.Dietitian 2.Medical social worker 3.Pain management clinic 4.Smoking cessation program

4.Smoking cessation program Rationale:Buerger's disease is a vascular occlusive disease that affects the medium and small arteries and veins. Smoking is highly detrimental to the client with Buerger's disease, so stopping smoking completely is recommended. Because smoking is a form of chemical dependency, referral to a smoking cessation program may be helpful for many clients. For many clients with Buerger's disease, symptoms are relieved or alleviated once smoking stops. A dietitian, a medical social worker, and a pain management clinic are not specifically associated with the lifestyle changes required in this disorder, although they may be needed if secondary problems arise.

A client has received antidysrhythmic therapy for the treatment of premature ventricular contractions (PVCs). The nurse evaluates this therapy as most effective if which finding is noted with regard to the PVCs? 1.They occur in pairs. 2.They appear to be multifocal. 3.They fall on the second half of the T wave. 4.They decrease to a frequency of less than 6 per minute.

4.They decrease to a frequency of less than 6 per minute. Rationale:PVCs are considered dangerous when they are frequent (more than 6 per minute), occur in pairs or couplets, are multifocal (multiform), or fall on the T wave. In each of these instances, the client's cardiac rhythm is likely to degenerate into ventricular tachycardia or ventricular fibrillation, both of which are potentially deadly dysrhythmias.


संबंधित स्टडी सेट्स

Chapter 10 test: internet law and privacy

View Set

What will you be doing next weekend?

View Set

我今年学十门课 Sentence structures

View Set

CHEM 211 Chapter 1 & 2 Homework questions

View Set